FNP Board review questions

अब Quizwiz के साथ अपने होमवर्क और परीक्षाओं को एस करें!

Match the statin with the correct potency 1) High-potency statin (50% reduction in LDL) 2) Medium-potency statin (30-49% reduction in LDL) 3) Low-potency statin (~ 20-30% reduction in LDL) A) Pravastatin (Pravachol) 10 mg B) Simvastatin (Zocor) 20 mg C) Rosuvastatin (Crestor) 20 mg D) Atorvastatin (Lipitor) 20 mg

1) High-potency = C) Rosuvastatin (Crestor) 20 mg, 40 mg, Atorvastatin (Lipitor) 40 mg 2) Medium-potency = B) Simvastatin (Zocor) 20 mg, D) Atorvastatin (Lipitor) 20 mg 3) Low-potency statin = A) Pravastatin (Pravachol) 10 mg

35-year old female with newly diagnosed impaired fasting glucose and Ā1C of 5.9% 1) What medication is first-line? 2) What if she is contemplating pregnancy?

1) Metformin 2) Metformin is OK in pregnancy

27-year old male with lupus has a eGFR of 48 and Ā1C of 7.2%, newly diagnosed with T2DM. 1) What medication is first choice in this patient?

1) Metformin (can be given as long as the GFT is > 45, monitor closely)

35-year old obese male with hx of T2DM is on the max dose of Metformin. His Ā1C is 7.6 (goal < 7%). He desires weight loss and is needle-phobic. 1) What medication class should be considered 2) Which med class(es) should be avoided?

1) SGLT-2 inhibitor (canagliflozin - Invokana, empagliflozin - Jardiance) 2)Sulfonylureas (a/w weight gain) GLP-1, insulin (injectables)

A 62-year old male is taking Metformin and has an Ā1C of 7.9% and is on a fixed budget. His Ā1C goal is < 7%. 1) What med class would be your first choice?

1) Sulfonylurea - lowers Ā1C by 1-2%

A 50-year old self-employed man drives a bread truck and takes max dose Metformin. He cannot tolerate hypoglycemia. His current Ā1C is 8.0 (goal < 7%). He has limited funds but has an affordable generic co-pay. 1) What medication would you start this patient on?

1) TZD (Actos or Avandia), DPP-4 (sitagliptin)?

Math the following diagnosis to the correct clinical presentation. 1. Complete abortion 2. Inevitable abortion 3. Threatened abortion 4. Incomplete abortion A) Uterine contents include a nonviable pregnancy that is in the process of being expelled. B) Some portion of the products of conception remains in the uterus, although the pregnancy is no longer viable. C) The products of conception have been completely expelled. D) Vaginal bleeding is present, but ultrasound evaluation shows a viable pregnancy.

1. Complete abortion = C) The products of conception have been completely expelled. 2. Inevitable abortion = A) Uterine contents include a nonviable pregnancy that is in the process of being expelled. 3. Threatened abortion = D) Vaginal bleeding is present, but ultrasound evaluation shows a viable pregnancy. 4. Incomplete abortion = B) Some portion of the products of conception remains in the uterus, although the pregnancy is no longer viable.

Match each sign with the correct characteristic 1. Hegar sign 2. Goodell sign 3. Chadwick sign A) blue-violet vaginal color B) softening of the uterus isthmus C) softening of the vaginal portion of the cervix

1. Hegar sign = B) softening of the uterus isthmus 2. Goodell sign = C) softening of the vaginal portion of the cervix 3. Chadwick sign = A) blue-violet vaginal color

Match the lab with the definition 1. Serum iron 2. Serum ferritin 3. Reticulocyte count 4. Peripheral smear A. A visual description of red blood cells B. Measure of iron in circulation C. Indicative of the bone marrow's ability to produce RBCs D. Measure of iron in storage

1. Serum Iron - B. Measure of iron in circulation 2. Serum ferritin - D. Measure of iron in storage 3. Reticulocyte Count - C. Indicative of the bone marrow's ability to produce RBCs 4. Peripheral Smear - A. A visual description of red blood cells

With an 0800 dose of the following insulin types, followed by inadequate dietary intake and/or excessive energy use, at what time would you expect hypoglycemia to most likely occur? A) Humalog (Lispro) B) Regular insulin (Humulin R, Novolin R) C) NPH insulin (Humulin N, Novolin N) D) Insulin glargine (Lantus)

A) Humalog (Lispro) at ~ 0830-0900 (short-acting, rapid onset insulin at peak onset of 30 minutes) B) Regular insulin (Humulin R, Novolin R) at ~1000-1100 (short acting insulin at peak onset of 2-3 hours) C) NPH insulin (Humulin N, Novolin N) at ~1400-2200 (intermediate-acting insulin at peak onset of 6 to 14 hours) D) Insulin glargine (Lantus) - hypoglycemia is unlikely because Lantus does not have a peak and duration of action is 24 hours

Mr. Johnson was started on amlodipine 5 mg for BP control 4 weeks ago in addition to his daily Lisinopril 20mg/HCTZ 25 mg daily. He also takes Advair 250/50 BID and albuterol PRN for asthma. 4 weeks after starting amlodipine, his average BP is 145/88 (goal < 140/90). He has no complaints. What is your next step? A) Increase amlodipine to 10 mg daily B) Add 25 mg HCTZ daily C) Add metoprolol 25 mg daily D) Continue on current medication regimen

A) Increase amlodipine to 10 mg daily

Which of the following statements best describes the Somogyi effect? A) Insulin-induced hypoglycemia triggers excess secretion of glucagon and cortisol, leading to hyperglycemia B) Early morning elevated blood glucose levels result in part from growth hormone and cortisol-triggering hepatic glucose release C) Late evening hyperglycemia is induced by inadequate insulin dose D) Episode of postprandial hypoglycemia occur as a result of inadequate food intake

A) Insulin-induced hypoglycemia triggers excess secretion of glucagon and cortisol, leading to hyperglycemia

A patient with asthma symptoms less than twice a week has A) Intermittent asthma B) Mild persistent asthma C) Moderate persistent asthma D) Severe persistent asthma

A) Intermittent asthma

Which of the following characteristics about atrial septal defect (ASD) is FALSE? A) It is more common in boys than girls B) Child presentation can range from entirely well to heart failure C) Full sports participation is typically acceptable with surgical correction D) Easy fatiguability is a sign of atrial septal defect

A) It is more common in boys than girls

Mrs. Smith is 85 years old and lives alone. She drives herself and participates in chair aerobics for 30 minutes, 3 days a week and does supervised resistance training twice a week. Her BP today is 160/70. BMI 23. PMHx includes extensive CAD with 2 stents placed 5 years ago, HTN, osteopenia, hyperlipidemia, GERD, hypothyroidism, and osteoarthritis. Meds: Pravastatin, Lisinopril 10 mg/HCTZ 12.5 mg daily, Aspirin 81 mg, Omeprazole 20 mg, Levothyroxine 25 mcg daily, Naproxen PRN for OA. Her labs reveal evidence of renal insufficiency ( Cr 1.6, eGFR 45 ml/min). What 4 medications in Mrs. Smith's regimen have the potential to diminish renal function? A) Lisinopril B) HCTZ C) Omeprazole D) Levothyroxine E) Naproxen F) Amlodipine G) Aspirin

A) Lisinopril B) HCTZ C) Omeprazole (PPI a/w acute kidney injury) E) Naproxen

A nurse practitioner auscultates a murmur at the apex of the heart. What diastolic murmur is likely? A) Mitral stenosis B) Aortic stenosis C) Mitral regurgitation D) Aortic regurgitation

A) Mitral stenosis

Which of the following should be the goal measurement in a person with DM and hypertension? A) SBP < 140 and DBP < 90 B) Ā1C equal to or greater than 7% C) Triglyceride level 200-300 D) HDL level 35-40

A) SBP < 140 and DBP < 90

Match the Suggested Ā1C goals for T2DM A) Patient with T1DM B) Most pregnant patients C) Older patients D) Most adults with T2DM 1. Ā1C < 7% 2. Ā1C < 6% 3. Ā1C < 8% 4. Ā1C < 6%

A - 2) Patients with T1DM = goal < 6% B - 4) Most pregnant patients = goal <6 % C - 3) Older patients = goal < 8% D - 1) Most adults with T2DM = goal < 7&

The NP sees a kindergartner with impetigo and advises the parents that she can return to school ________ hours after starting effective antimicrobial therapy. A) 24 B) 48 C) 72 D) 96

A) 24 hours

Based on the primary mechanism of action of Metformin, what lab value must be evaluated and monitored? A) ALT, AST B) BUN, Cre C) eGFR D) Potassium

A) AST, ALT Metformin works in the liver to decrease hepatic glucose output

Mr. Jones is a 66-year old black man. He is a retired postal worker. PMHx includes HTN, a-fib, diabetes, HLD, gout, BPH, osteoarthritis in his knees. He quit smoking 5 years ago. His BP today is 160/90, HR 84. BMI 32. Meds: Irbesartan 150 mg/HCTZ 12.5 mg, Tamulosin (Flomax) 0.8 mg daily, amlodipine 5 mg daily, metoprolol 50 mg daily, aspirin 81 mg daily, naproxen 500 mg PRN for knee pain, glipizide ER 5 mg (inconsistently - it sometimes makes him feel "weak") What are the 4 most important risk factors for abdominal aortic aneurysm in Mr. Jones? A) Age B) Smoking history C) Male gender D) Race E) Family history F) Presence of a-fib G) Hyperlipidemia

A) Age > 65 B) Smoking history C) Male gender = 5-8x more common in males E) Family history

Mr. Johnson is a 72-year old newly diagnosed Type 2 diabetic. Which of the following characteristics increase the likelihood of lactic acidosis? Select 4. A) Concurrent use of contrast dye B) eGFR > 45 C) Dehydration D) Presence of heart failure E) Presence of binge drinking F) Use of CCBs G) Presence of BPH

A) Concurrent use of contrast dye C) Dehydration D) Presence of heart failure E) Presence of binge drinking

What are the two most common side effects of Metformin? A) Diarrhea B) Weight gain C) Headaches D) Flatulence

A) Diarrhea D) Flatulence

Elevated inhibin-A is noted when a pregnant woman is at an increased risk of having an infant with A) Down syndrome B) Edwards syndrome C) open neural tube defect D) hemolytic anemia

A) Down syndrome Decreased AFP, decreased estriol, increased hcG and inhibin-A

A 63-year old patient w/ COPD c/o a pounding heart after using his inhaler. Which of the following is the least likely culprit? A) Fluticasone B) Albuterol C) Iptratropium D) Salmeterol

A) Fluticasone

A 23-year old female presents with mild mitral stenosis and is asymptomatic. Which of the following is correct concerning her participation in sports? A) Full activity is likely acceptable B) Prolonged aerobic exercise should be avoided C) An ACE inhibitor should be prescribed prior to participation D) Participation should be limited to non-contact sports

A) Full activity is likely acceptable

A 19-year old man presents with stage 1 HTN. Which of the following is correct concerning his participation in sport activities? A) Full activity should be encouraged B) Weight lifting is contraindicated C) He should have an exercise tolerance test D) A beta-adrenergic antagonist should be prescribed

A) Full activity should be encouraged

A 25-year old woman presents with sinus arrhythmia. Which of the following statements is correct concerning her participation in sport activities? A) Full activity should be encouraged B) Weight lifting is contraindicated C) She should have an exercise tolerance test D) a calcium channel antagonist should be prescribed

A) Full activity should be encouraged Sinus arrhythmia in a younger adult is a normal finding

What is the most common adverse effect noted with alpha-glucosidase inhibitor use (e.g. acarbose [Precose])? A) GI upset B) hepatotoxicity C) renal impairment D) symptomatic hypoglycemia

A) GI upset

Which of the following characteristics apply to T2DM? A) Heredity and obesity are major risk factors B) Pear-shaped body type is common C) Exogenous insulin is needed for control of disease D) Physical activity enhances insulin resistance

A) Heredity and obesity are major risk factors

Which of the following statements best describes the current best practice regarding the use of sliding scale insulin to treat elevated blood glucose? A) The use of sliding scale therapy is discouraged because this method treats hyperglycemia after it as has already occurred B) Sliding scale insulin in response to elevated glucose is a safe and helpful method of treating hyperglycemia C) Delivering insulin in this manner is acceptable within the acute care setting only D) The use of sliding scale insulin is appropriate in treating T1DM only

A) The use of sliding scale therapy is discouraged because this method treats hyperglycemia after it as has already occurred

How do inhaled betá-agonists work to treat shortness of breath in COPD? A) They cause bronchodilation in the lungs B) They block the action of acetylcholine and prevent bronchoconstriction

A) They cause bronchodilation in the lungs

Which of the following statements regarding vitamin D is false? A) Vit D diminishes secretion of insulin B) Vit D inhibits abnormal cellular growth C) Vit D facilitates the absorption of calcium and phosphorous D) Vit D reduces inflammation

A) Vit D diminishes secretion of insulin Vitamin D stimulates insulin secretion in response to increased insulin demands and plays a role in the maintenance of normoglycemia.

Which of the following patients with acute gouty arthritis is the best candidate for local corticosteroid injection? A) a 66-year old with a gastric ulcer B) a 44-year old taking a thiazide diuretic C) a 68-year old with T2DM D) a 32-year old binge drinker

A) a 66-year old with a gastric ulcer

The mechanism of action of the DPP-4 inhibitors (e.g. sitagliptin [Januvia]) is as A) a drug that increases levels of incretin, increasing synthesis and release of insulin from pancreatic beta cells B) a product virtually identical in action to sulfonylureas C) a drug that increases insulin action in the peripheral tissues and reduces hepatic glucose production D) a facilitator of renal glucose excretion

A) a drug that increases levels of incretin, increasing synthesis and release of insulin from pancreatic beta cells

First-line pharmacological intervention for milder OA should be a trial of A) acetaminophen B) tramadol C) celecoxib D) intraarticular corticosteroid injection

A) acetaminophen

An oral antimicrobial option for the treatment of methicillin-sensitive S. aureus includes all of the following except A) amoxicillin B) dicloxacillin C) cephalexin D) cefadroxil

A) amoxicillin

During a preparticipation sports exam, you hear a grade 2/6 midsystolic ejection murmur, heard best at the second intercostal space of the left sternal border, in an asymptomatic young adult. The murmur disappears with position change from supine to standing. This most likely represents A) an innocent flow murmur B) mitral valve incompetency C) aortic regurgitation D) Mitral valve prolapse

A) an innocent flow murmur (physiologic murmur)

Long term bisphosphonate treatment (e.g. > 5 years) has been associated with A) atypical fractures B) hyperprolactinemia C) osteoarthritis D) bone marrow suppression

A) atypical fractures Bisphosphonates (alendronate [Fosamax], ibandronate [Boniva], risedronate [Actonel], zoledronic acid [Reclast]) inhibit the resorptive activity of osteoclasts and can help to modestly increase bone mass and reduce fracture risk *Take at least 30 minutes before food, liquids, or other meds and remain upright for at least 1 hour to reduce the risk of drug-induced esophagitis

SSRI withdrawal syndrome is best characterized as A) bothersome but not life-threatening B) potentially life-threatening C) most often seen with medications with a longer half life D) associated with seizure risk

A) bothersome but not life-threatening

First-line therapy for prepatellar bursitis should include A) bursal aspiration B) intrabursal corticosteroid injection C) acetaminophen D) knee splinting

A) bursal aspiration

You have initiated acne treatment for an 18-year old male by prescribing doxycycline. He returns to the clinic 3 weeks later and complains that his skin is no better. Your next action is to A) counsel him that 6 to 8 weeks of treatment is often needed before significant improvement is achieved B) discontinue doxycycline and start minocycline therapy C) advise him that antibiotics are not likely an effective treatment for him and should not be continued D) add a second antimicrobial agent such as TMP-SMX

A) counsel him that 6 to 8 weeks of treatment is often needed before significant improvement is achieved

One of the most common trigger agents for contact dermatitis is A) exposure to nickel B) use of fabric softener C) bathing with liquid body wash D) eating spicy foods

A) exposure to nickel

You consider prescribing insulin glargine (Lantus) because of its A) extended duration of action B) rapid onset of action C) ability to prevent diabetic end-organ damage D) ability to preserve pancreatic function

A) extended duration of action

You see an obese 25-year male patient with acanthosis nigricans and consider ordering A) fasting blood glucose B) LFTs C) RPR D) ESR

A) fasting blood glucse

A Tzanck smear that is positive for giant multinucleated cells was taken from a lesion caused by A) herpes virus B) S. aureus C) streptococci D) an allergic reaction

A) herpes virus

Which of the following is an unlikely consequence of untreated metabolic syndrome and insulin resistance in a woman of reproductive age? A) hyperovulation B) irregular menses C) acne D) hirsutism

A) hyperovulation

Which of the following are interventions for microalbuminuria in a patient with DM? (select all that apply) A) improved glycemic control B) strict dyslipidemia control C) use of an optimized dose of an ACEI or ARB D) the use of an a ACEI or ARB

A) improved glycemic control B) strict dyslipidemia control C) use of an optimized dose of an ACEI or ARB

Patients with gluteus medius or deep trochanteric bursitis typically present with A) increased pain from resisted hip abduction B) limited hip ROM C) sciatic nerve pain D) heat over the affected area

A) increased pain from resisted hip abduction

The most common form of acquired anemia during pregnancy is A) iron deficiency B) folate deficiency C) thalassemia D) vitamin B12 deficiency

A) iron deficiency Usually because the woman enters pregnancy with iron deficiency, rather than developing this problem d/t increased iron requirements

After use, the onset of action of lispro (Humalog) occurs in A) less than 30 minutes B) approximately 1 hour C) 1-2 hours D) 3-4 hours

A) less than 30 minutes

Which of the following medications contributes to the development of acne vulgaris? A) lithium B) propranolol C) sertraline D) clonidine

A) lithium

Risk factors for preeclampsia include all of the followingn except A) low maternal weight B) age younger than 16 or older than 40 C) collagen vascular disease D) first pregnancy with a new partner

A) low maternal weight

Risk factors for acute gouty arthritis include A) obesity B) female gender C) rheumatoid arthritis D) joint trauma

A) obesity

You are examining a 17-year old male with acne and notice blackheads and whiteheads. What are the medical terms for blackheads and whiteheads, respectively? A) open comedomes and closed comedomes B) closed follicles and open follicles C) closed comedomes and open comedomes D) open follicles and closed follicles

A) open comedomes (blackheads) and closed comedomes (whiteheads)

Clinical presentation of T1DM usually includes all of the following, except A) report of recent unintentional weight gain B) ketosis C) thirst D) polyphagia

A) report of recent unintentional weight gain

Which of the following characteristics apply to T1DM? A) significant hyperglycemia and ketoacidosis results from lack of insulin B)T1DM is commonly diagnosed on routine exam or workup for other health problems C) Initial response to oral sulfonylureas is usually favorable D) Insulin resistance is a significant part of the disease

A) significant hyperglycemia and ketoacidosis results from lack of insulin

Recommended exercises for patients with OA of the knee include all of the following except A) squatting with light weights B) straight-leg raises without weights C) quadriceps sets D) limited weight-bearing aerobic exercises

A) squatting with light weights

Patients with olecranon bursitis typically present with A) swelling and redness over the affected area B) limited elbow range of motion C) nerve impingement D) destruction of the joint space

A) swelling and redness over the affected area

If prescribing itraconazole (Sporanox), the NP should consider that A) the drug is a cytochrome P-450 3À4 inhibitor B)one pulse cycle is recommended for fingernail treatment, and two cycles are needed for toenail treatment C) continuous therapy is preferred in the presence of liver disease D) taking the drug on an empty stomach increases the efficacy

A) the drug is a cytochrome P-450 3À4 inhibitor (Fluconazole is a CYP450 2CP inhibitor; Terbinafine has signficantly less drug interaction potential) An example of pulse dosing for itraconazole is 400 mg daily for the first week of the month x2 months for fingernails and x3 months for toenails. The product is held in the nail matrix for months after therapy, and pulse therapy reduces risk of hepatic enzyme elevation.

The "quad screen" is used to help detect increased risk for which of the following conditions in the fetus? A) trisomy 21 and open neural tube defects B) cystic fibrosis and Angelman syndrome C) Tay-Sachs disease and trisomy 18 D) sick cell anemia and beta-thalassemia major

A) trisomy 21 and open neural tube defects

In counseling a patient with seborrheic dermatitis on the scalp about efforts to clear the lesions, you advise her to A) use ketoconazole shampoo B) apply petroleum jelly nightly to the affected area C) coat the area with a high-potency steroid cream three times a week D) expose the lesions to heat periodically by carefully using a hair dryer

A) use ketoconazole shampoo

Changes to the joint during OA can typically include all of the following except A) widening of the joint space B) articular cartilage wears away C) formation of bone spurs D) synovial membrane thickness

A) widening of the joint space

Which of the following best describes the presentation of a patient with OA? A) worst symptoms in the weight-bearing joints later in the day B) symmetrical early morning stiffness C) sausage-shaped digits with associated skin lesions D) back pain with rest and anterior uveitis

A) worst symptoms in the weight-bearing joints later in the day

Clinical conditions with a presentation similar to acute bursitis include ____________. (select all that apply) A) rheumatoid arthritis B) septic arthritis C) joint trauma D) pseudogout

A, B, C, & D

Which of the following best describe the physical activity recommendations such as brisk walking for a 55-year old female with newly diagnosed T2DM? (Select all that apply) A) The goal should be for a total increased physical activity of at least 150 minutes per week B) Increased physical activity is recommended for at least 30 minutes/day, at least 3 times per week, with no more than 48 hours without exercise C) Some form of resistance exercises, such as lifting dumbells or using an exercise band should be included at least 3 times a week D) Vigorous aerobic or resistance activity is potentially contraindicated in the presence of proliferative or severe nonproliferative retinopathy due to the risk of vitreous hemorrhage or retinal detachment

A, B, C, D

Mr. Jones (a 66-year old black man. He is a retired postal worker. PMHx includes HTN, a-fib, diabetes, HLD, gout, BPH, osteoarthritis in his knees) has had 2 episodes of heart failure in the past 12 months, one that required in-patient hospitalization. Meds: Irbesartan 300 mg daily, HCTZ 25 mg daily, Furosemide 40 mg daily PRN 2-lb weight gain in 2 days, metoprolol 100 mg daily, pravastatin 40 mg daily, glipizide 5 mg/500mg metformin daily, flomax daily, aspirin, naproxen Which of Mr. Jones' medications are most beneficial for treatment of his heart failure? Select all that apply A) Irbesartan 300 mg daily B) HCTZ 25 mg C) Furosemide 40 mg daily PRN 2-lb weight gain in 2 days D) Metoprolol 100 mg daily E) Pravastatin 40 mg daily F) Glipizide 5 mg/Metformin 500 mg daily G) Flomax daily H) Aspirin, naproxen

A, B, C, D A) Irbesartan 300 mg daily = increases cardiac output B) HCTZ 25 mg = mild diuretic decreases workload on the heart C) Furosemide 40 mg daily PRN 2-lb weight gain in 2 days = more potent diuretic D) Metoprolol 100 mg daily = beta blockers decrease morbidity/mortality in CHF

Which statements are true about placenta previa? Select all that apply. A) Painless vaginal bleeding is typical. B) This usually occurs in the 2nd trimester. C) No referral is needed unless it worsens. D) This may progress to placenta abruptio.

A, B, D A) Painless vaginal bleeding is typical. B) This usually occurs in the 2nd trimester. D) This may progress to placenta abruptio. Also, a vaginal exam should NOT be performed if placenta previa is suspected.

The ADA recommends annual screening for diabetes for patients with a BMI greater than _______ and one ore more risk factors.

25

Which of the following findings can be found in a person with stage 2 Lyme disease? A) peripheral neuropathic symptoms B) AV heart block C) conductive hearing loss D) macrocytic anemia

B) AV heart block Stage 2 Lyme disease = early disseminated infection, typically months after the initial infection. The classic rash may reappear with multiple lesions, usually accompanied by arthralgias, myalgia, headache, and fatigue. Cardiac manifestations such as heart block, neurologic findings such as Bell's palsy, and aseptic meningitis are less common.

Which of the following best describes ethnicity and insulin sensitivity? A) Little variation exists in insulin sensitivity among different ethnic groups B) African Americans are typically less sensitive to the effects of insulin when compared to people of European ancestry C) Mexican Americans are likely the most insulin sensitive ethnic group residing in North America D) The degree of insulin sensitivity has little influence on insulin production

B) African Americans are typically less sensitive to the effects of insulin when compared to people of European ancestry

A 64-year male with seborrheic dermatitis mentions that his conditions seems to get better in the summer "when I'm outside more" and much worse in the winter. The NP responds A) Sun exposure is a recommended therapy to treat this condition. B) Although sun exposure is noted to improve the skin lesions associated with seborrheic dermatitis, it is potentially associated with an increased rate of skin cancer. C) The decreased humidity in the summer months observed in many areas of North America contributes to the improvement in seborrheic lesions. D) Use a high-potency topical steroid during the winter months and taper off in the summer.

B) Although sun exposure is noted to improve the skin lesions associated with seborrheic dermatitis, it is potentially associated with an increased rate of skin cancer.

A 70-year old house painter reports a 4-week history of exertional dyspnea, chest tightness, and cough for the past 3 months. He has never smoked. What diagnoses are included in your differential? Select 4. A) Asthma B) Angina C) COPD D) GERD E) Pneumonia F) Tuberculosis G) Heart Failure

B) Angina C) COPD F) Tuberculosis G) Heart failure

______________ medications are among the most commonly used medications by women in the first trimester of pregnancy. A) Antiepileptic drugs B) Antibiotics C) Antihypertensives D) Opioids

B) Antibiotics

If a 19-year old college female diagnosed with community acquired pneumonia was pregnant in her first trimester, how could she be managed? A) Levofloxacin 750 mg PO daily x 5 days B) Azithromycin 500 mg on day 1, then 250 mg daily on days 2-5, plus amoxicillin 1000 mg BID C) Cephalexin 500 mg PO BID x 5 days D) Doxycyline 100 mg PO BID x 5 days

B) Azithromycin 500 mg on day 1, then 250 mg daily on days 2-5, plus amoxicillin 1000 mg BID Quinolones (e.g. levofloxacin) and tetracyclines (e.g. doxycycline) are teratogenic and should not be given during pregnancy

Deformity of the proximal interphalangeal joints found in elderly patients with OA is known as A) Heberden nodes B) Bouchard nodes C) hallus valgus D) Dupuytren contracture

B) Bouchard nodes

What are the most common side effects of long-term inhaled steroid use? A) Osteoporosis and GERD B) Cataracts and osteopenia C) Hyperkalemia and diabetes D) Hypertension and diabetes

B) Cataracts and osteopenia

A 58-year old man has the following lab results. TSH 13.5 (normal 0.4-4.8), T4 1.1 (norm 0.8-1.8) 6 weeks later: TSH 15.6, T4 0.9 Select the best answer. A) Diagnose as primary hypothyroidism, treat with levothyroxine B) Diagnose as subclinical hypothyroidism, treat with levothyroxine C) Diagnose as subclinical hypothyroidism, do not treat D) Diagnose as subclinical hyperthyroidism, treat with methimazole

B) Diagnose as subclinical hypothyroidism, treat with levothyroxine Treatment of subclinical hypothyroidism is controversial.... If patient has subclinical hypothyroidism with TSH > 10 --> Treat to PREVENT conversion to primary hypothyroidism If TSH < 10 --> monitor. Associated with risk of afib (immediate risk), accelarated bone loss (long-term risk, leeches calcium and vit D from the bones)

A 42-year old data entry clerk has asymptomatic aortic regurgitation. What would you expect in listening to the murmur? A) A late diastolic click near the apex B) Diastolic sounds heard loudest near the 2nd intercostal space, right sternal border C) Diastolic sounds heard in the midsternal area D) Intermittent diastolic murmur heard near the left sternal border

B) Diastolic sounds heard loudest near the 2nd intercostal space, right sternal border

Which of the following is the most common UTI organism in pregnant women? A) Pseudomonas aeruginosa B) Escherichia coli C) Klebsiella pneumoniae D) Proteus mirabilis

B) Escherichia coli

Mr. Johnson is a 58-year old male with T2DM. He currently uses a single 10 unit daily dose of long-acting insulin glargine (Lantus). His fasting blood glucose has been between 141 to 180. Which of the following best describes the next step in his therapy? A) Continue on his current dose B) Increase his Lantus by 4 units per day C) Increase his Lantus by 1 unit per day D) Increase his Lantus by 6 units per day

B) Increase his Lantus by 4 units per day

A patient with asthma symptoms more than twice a week, but not daily with occasional nighttime awakenings has A) Intermittent asthma B) Mild persistent asthma C) Moderate persistent asthma D) Severe persistent asthma

B) Mild persistent asthma

Which of the following statements regarding Edwards syndrome is true? A) Edwards syndrome is more common than Down syndrome. B) Most affected infants with Edwards syndrome die during the first year of life. C) Edwards syndrome is unlikely to cause developmental disability. D) Edwards syndrome is associated with elevated AFP.

B) Most affected infants with Edwards syndrome die during the first year of life.

A 38-year old mother of two teenagers recently recovered from Mycoplasma pneumonia a couple of weeks ago. She asks if she should get the "pneumonia shot." She takes levothyroxine 88 mcg daily for hypothyroidism, but is otherwise healthy. How do you respond? A) No, it's too soon after your infection B) No, it's not indicated C) Yes, you can get it in about a month D) Yes you can get it today

B) No, it's not indicated An otherwise healthy adult without immunocompromise or multiple comorbid conditions is not a "vulnerable population" The pneumonia vaccine does not prevent mycoplasma pneumonia

Which of the following statements is false regarding sunlight exposure and vitamin D production? A) In the continental US, summertime exposure to sunlight can produce the majority of the body's requirement for vitamin D B) One glass of fortified milk has an equivalent amount of vitamin D to what is produced after 10 minutes of sun exposure in the summertime in a healthy, young individual C) Use of sunscreen can block the majority of sun-induced vitamin D production D) A person with a darker skin tone produces less vitamin D from sun exposure than a person with a lighter skin tone

B) One glass of fortified milk has an equivalent amount of vitamin D to what is produced after 10 minutes of sun exposure in the summertime in a healthy, young individual

A common infective agent in domestic pet cat bites is A) viridans streptococcus species B) Pasteurella multocida C) Bateroides species D) Haemophilus influenzae

B) Pasteurella multocida

In addition to acanthosis nigricans, which of the following is a risk factor for insulin resistance? A) Delivery of infant > 9 lbs B) Severe obesity C) Age > 65 D) Sleep Apnea

B) Severe obesity

A pregnant mother who is Rh negative had a suspected miscarriage early this morning. Her LMP was 8 weeks ago. Which of the following statements about administration of Rhogam (anti-D immune globulin) is correct? A) She does not need Rhogam B) She should receive Rhogam now C) She only needs Rhogam if miscarriage occurs in the 2nd trimester D) She only needs Rhogam if miscarriage occurs in the 3rd trimester

B) She should receive Rhogam now

A 75-year old patient was diagnosed with hypothyroidism today. Her calculated levothyroxine replacement is 88 mcg daily. How shoulder her levothyroxine dose be managed? A) Refer to endocrinology B) Start Levothyroxine 25 mcg daily C) Start Levothyroxine 44 mcg daily D) Start Levothyroxine 50 mcg daily

B) Start Levothyroxine 25 mcg daily Older patients and those with underlying cardiac issues or multiple comorbidites should start low and titrate slowly

How do inhaled anticholinergics work to treat shortness of breath in COPD? A) They cause bronchodilation in the lungs B) They block the action of acetylcholine and prevent bronchoconstriction

B) They block the action of acetylcholine and prevent bronchconstriction

Who is the best candidate for isotretinoin (Accutane) therapy? A) a 17-year old patient w/ pustular lesions and poor response to benzoyl peroxide B) a 20-year old patient with cystic lesions who has tried various therapies with minimal improvement C) a 14-year old patient with open and closed comedomes and a family history of "ice pick" scars D) an 18-year old patient with inflammatory lesions and improvement with tretinoin (Retin-A)

B) a 20-year old patient with cystic lesions who has tried various therapies with minimal improvement

Which of the following patients would be an appropriate candidate for treatment with Forteo (teriparatide)? A) a 54-year old female with osteopenia B) a 65-year old female with a BMD T-score of -2.5 and a previous hip fracture C) a 67-year old female with a BMD T-score of -1 D) a 72-year old female with a stable BMD T-score of -1.5 with bisphosphonate treatment for the past 3 years

B) a 65-year old female with a BMD T-score of -2.5 and a previous hip fracture Teriparatide is a bone-forming (anabolic) medication (parathyroid hormone). It can reduce the risk of vertebral fractures by 65% in patients with osteoporosis. It is typically reserved for women with very low BMD or who have had a prior fracture. Use of teriparatide is limited to 2-year duration due to a potential increased risk of osteosarcoma.

The mechanism of action of sulfonylureas is as A) an antagonist of insulin receptor site activity B) a product that enhances insulin release C) a facilitator of renal glucose excretion D) an agent that can reduce hepatic glucose production

B) a product that enhances insulin release

You see a 67-year old woman who has been treated for pain due to OA of the hip for the past 6 months and who asks about hip replacement surgery. She complains of pain even at night when sleeping and avoids walking even moderate distances unless absolutely necessary. In counseling the patient, you mention all of the following except A) arthroplasty can be considered when pain is not adequately controlled B) arthroplasty is not needed if the patient can walk even short distances C) arthroplasty candidates must be able to tolerate a long surgical procedure D) rehabilitation following surgery is essential to achieve maximal function of the joint

B) arthroplasty is not needed if the patient can walk even short distances

Hemoglobin Ā1C should be tested A) at least annually for all patients B) at least twice a year in patients who are meeting treatment goals and have stable glycemic control C) monthly in patients whose therapy has changed or who are not meeting glycemic goals D) only via standardized lab testing due to inaccuracies associated with point-of-care testing

B) at least twice a year in patients who are meeting treatment goals and have stable glycemic control

An adverse effect associated with the use of glucosamine is A) elevated ALT and AST B) bronchospasm C) increased bleeding risk D) QT prolongation

B) bronchospasm

When treating a woman who is 28-weeks pregnant for a UTI, which antibiotic would you consider prescribing? A) trimethoprim-sulfamethoxazole (TMP-SMX) B) cephalexin C) ciprofloxacin D) doxycycline

B) cephalexin Treatment options for asymptomatic bacteruria and symptomatic UTI include beta lactams (e.g. amoxicillin, cephalexin, cefpodoxime, cefixime, and amoxicillin/clavulanate) and nitrofurantoin *Nitrofurantoin should be avoided after the 36th week of gestation because of potential risk for hemolysis if the fetus is G6PD deficient

Which of the Which of the following is the most potent topical corticosteroid? A) mometasone furoate 0.1% (Elocon ointment) B) clobetasol propionate 0.05% (Cormax) C) hydrocortisone 2.5% D) fluocinonide 0.05% (Lidex)

B) clobetasol propionate 0.05% (Cormax) = SUPER HIGH potency hydrocortisone 2.5 % = LOW potency mometasone furoate 0.1% = MIDRANGE potency fluocinonide 0.05% (Lidex) = HIGH potency

Initial treatment of lateral epicondylitis includes all of the following except A) rest and activity modifications B) corticosteroid injections C) topical or oral NSAIDs D) counterfource bracing

B) corticosteroid injections

Mr. Johnson is a 72-year old newly diagnosed Type 2 diabetic. Which of the choices below must be considered prior to starting Metformin in this patient? Select 4. A) CBC B) eGFR C) LFTs D) Presence of heart failure E) Presence of binge drinking F) Use of CCBs G) Presence of BPH

B) eGFR C) LFTs D) Presence of heart failure E) Presence of binge drinking

Which of the following medications should be used with caution in a person with a known or suspected sulfa allergy? A) metformin B) glyburide C) rosglitazone D) NPH insulin

B) glyburide

Risk factors for medial epicondylitis include playing A) tennis B) golf C) baseball D) volleyball

B) golf

Which of the following best describes seborrheic dermatitis? A) flaky lesions in the antecubital and popliteal spaces B) greasy, scaling lesions in the nasolabial folds C) intensely itchy lesions in the groin folds D) silvery lesions on the elbow and knees

B) greasy, scaling lesions in the nasolabial folds

The likely causative organism of nonbullous impetigo in a 6-year old child include A) H. influenzae and S. pneumoniae B) group A streptococcus and S. aureus C) M. cararrhalis and select viruses D) P. aeruginosa and select fungi

B) group A streptococcus and S. aureus

A Still murmur A) is an indication to restrict sports participation selectively B) has a buzzing quality C) is usually heard in patients who experience dizziness when exercising D) is a sign of cardiac structural abnormality

B) has a buzzing quality -also known as a "vibratory innocent murmur," a benign finding

You are examining an 18-year old male who is seeking a sports clearance physical. You note a mid-systolic murmur that gets louder when he stands. This is most likely A) aortic stenosis B) hypertrophic cardiomyopathy C) a physiologic murmur D) a Still murmur

B) hypertrophic cardiomyopathy

The most common cause of acute bursitis is A) inactivity B) joint overuse C) fibromyalgia D) bacterial infection

B) joint overuse

First-line therapy for treating patients with acute gouty arthritis usually includes A) aspirin B) naproxen sodium C) allopurinol D) probenecid

B) naproxen sodium

The International Diabetes Federation's diagnostic criteria for metabolic syndrome include: A) an obligatory finding of persistent hyperglycemia B) notation of ethnic-specific waist circumference measurements C) documentation of microalbuminuria D) a family history of T2DM

B) notation of ethnic-specific waist circumference measurements

You hear a fixed split second heart sound (S2) in a 28-year old female who wants to start an exercise program and consider that it is A) a normal finding in younger adults B) occasionally found in uncorrected atrial septal defect C) the result of valvular sclerosis D) often found in patients with right bundle branch block

B) occasionally found in uncorrected atrial septal defect

The use of exenatide has been associated with the development of ____________. A) leukopenia B) pancreatitis C) lymphoma D) vitiligo

B) pancreatitis

Medications commonly pass through the placenta via A) facilitated transport B) passive diffusion C) capillary pump action D) mechanical carrier state

B) passive diffusion

Dietary recommendations for a person with gouty arthritis include avoiding foods high in A) artificial flavors and colors B) purine C) vitamin C D) protein

B) purine

Risk factors for mitral regurgitation include a prior diagnosis of all of the following except A) rheumatic heart disease B) scarlet fever C) endocarditis D) calcific annulus

B) scarlet fever

Maternal iron requirements are greatest during what part of pregnancy? A) first trimester B) second and third trimesters C) equal throughout pregnancy D) preconception

B) second and third trimesters Maternal iron requirements increase in the 2nd and 3rd TM in part due to the fetus's need to build iron stores

The spectrum of antimicrobial activity of mupirocin (Bactroban) includes A) primarily gram-negative organisms B) select gram-positive organisms C) Pseudomonas species and anerobic organisms D) only organisms that do not produce beta-lactamase

B) select gram-positive organisms

Risk factors for the development of postherpetic neuralgia include A) Age < 50 years old at the time of the outbreak B) severe prodomal symptoms C) lumbar dermatomal distribution D) low volume of lesions

B) severe prodomal symptoms

The use of all of the following medications can trigger gout except A) aspirin B) statins C) diuretics D) niacin

B) statins Aspirin, diuretics, and niacin can precipitate gout by causing hyperuricemia

A common site for atopic dermatitis in an infant is A) the diaper area B) the face C) the neck D) the posterior trunk

B) the face

The clinical presentation of acute gouty arthritis affecting the base of the great toe includes A) slow onset of discomfort over many days B) the greatest swelling and pain along the median aspect of the joint C) improvement of symptoms with joint rest D) fever

B) the greatest swelling and pain along the median aspect of the joint

The components of HELLP syndrome include all of the following except A) elevated hepatic enzymes B) thrombocytosis C) hemolysis D) eclampsia

B) thrombocytosis Preeclampsia can progress to the syndrome of hemolysis (with resulting anemia), elevated liver enzymes , low platelets and eclampsia.

First-line treatment of impetigo with less than 5 lesions of 1-2 cm in diameter on the legs in a 9-year old female is A) topical neomycin B) topical mupirocin C) oral cefixime D) oral doxycycline

B) topical mupirocin

Which of the following is recommended to prevent a burn wound infection? A) topical corticosteroid B) topical silver sulfadiazine C) oral erythromycin D) oral moxifloxacin

B) topical silver sulfadiazine

Which prescribing tretinoin (Retin-A), the NP advises the patient to A) use with an astringent to minimize skin irritation B) use with sunscreen because the drug is photosensitizing C) expect a significant improvement in acne lesions after approximately 1 week of use D) if no improvement is noted in a few days, double the amount of medication used on the skin

B) use with sunscreen because the drug is photosensitizing

Clinical features of bullous impetigo include A) intense itching B) vesicular lesions C) dermatomal pattern D) systemic symptoms (e.g. fevers/chills)

B) vesicular lesions

Postpartum "baby blues" typically begin A) 1-2 weeks before delivery B) within a few days after delivery C) 1-2 weeks after delivery D) approximately 1 month after delivery

B) within a few days after delivery Postpartum depression can occur in ~ 10- 20% of new mothers and usually presents around 2 to 4 months after birth. S/Sx are more intense than "baby blues" and last longer.

What drugs could you use to manage hypertension in women who are pregnant or contemplating pregnancy? Select all that apply. A) Lisinopil B) Methyldopa C) Nifedipine D) Labetolol E) Losartan F) Verapamil G) Hydralazine H) Diltiazem

B, C, D, F, G, & H B) Methyldopa C) Nifedipine D) Labetolol F) Verapamil G) Hydralazine H) Diltiazem CCBs (nifedipine, diltiazem, and verapamil), beta blockers, Methyldopa, Hydralazine are safe in pregnancy AVOID ACEIs and ARBs in pregnancy - they affect the developing kidneys and lead to miscarriage.

A 45-year old patient has subclinical hypothyroidism. Her TSH is 6.2. What are the major risks of prescribing levothyroxine? A) There are very few risks B) Hyperlipidemia, atrial fibrillation C) Accelerated bone loss, atrial fibrillation D) She may develop hypertension and tachycardia

C) Accelerated bone loss, atrial fibrillation

A preparticipation physical screening exam should include A) CBC B) UA C) BP measurement D) spine xray

C) BP measurement

A 78-year old female how is a retired school teacher has been diagnosed with mitral stenosis. What would you expect in listening to the murmur? A) A late systolic click near the apex B) Radiation of soft diastolic sounds into the neck/carotid arteries C) Diastolic sounds heard loudest at the apex D) Soft diastolic sounds in the left mid-clavicular area

C) Diastolic sounds heard loudest at the apex

Which of the following statements regarding Down syndrome is true? A) Most infants affected with Down syndrome are born to women older than 35 years of age. B) Down syndrome is noted in about 1 in 10,000 live births. C) Down syndrome is associated with decreased maternal serum AFP level. D) Antenatal serum analysis is sufficient to make the diagnosis of Down syndrome.

C) Down syndrome is associated with decreased maternal serum AFP level. The risk of Down syndrome is ~1 in 1000 live births (risk increases to 1 in 270 in women 35 and older, 1 in 100 in women 40 and older)

The most helpful diagnostic test to perform during acute gouty arthritis is A) ESR B) serum uric acid C) analysis of aspirate from the affected joint D) joint x-ray

C) analysis of aspirate from the affected joint

Secondary gout can be caused by all of the following conditions except A) psoriasis B) hemolytic anemia C) bacterial cellulitis D) renal failure

C) bacterial cellulitis psoriasis increases catabolism and purine turnover renal failure decreases uric acid clearance

First-line treatment for acne vulgaris with closed comedomes includes A) oral antibiotics B) isotretinoin C) benzoyl peroxide D) hydrocortisone cream

C) benzoyl peroxide

Which of the following dietary supplements is associated with increased risk for gout? A) vitamin A B) gingko biloba C) brewer's yeast D) glucosamine

C) brewer's yeast

A 72-year old man presents at an early stage of OA in his left knee. He mentions that he heard about the benefits of using glucosamine and chondroitin for treating joint problems. In consulting the patient, you mention all of the following except A) any benefit can take at least 3 months of consistent use before observed B) glucosamine is not associated with an drug interactions C) clinical studies have consistently shown benefit of long-term use of glucosamine and chondroitin for treating OA of the knee D) chrondroitin should be used with caution because of its antiplatelet effect

C) clinical studies have consistently shown benefit of long-term use of glucosamine and chondroitin for treating OA of the knee

An impetigo lesion that becomes deeply ulcerated is known as A) cellulitis B) erysipelas C) ecthyma D) empyema

C) ecthyma

During type I hypersensitivity reactions, histamine released from degraded mast cells causes all of the following except A) vasodilation B) mucous gland stimulation C) enhanced sebum production D) tissue swelling

C) enhanced sebum production

The NP writes a prescription for a topical agent and anticipates the greatest rate of absorption when it is applied to the ____________. A) palms of the hands B) soles of the feet C) face D) abdomen

C) face

The typical patient with primary hypothyroidism has: A) skin, hair, and nail changes B) an abnormal lipid panel C) failure of the thyroid gland D) involvement of the pituitary-hypothalamic axis

C) failure of the thyroid gland - PRIMARY hypothyroidism (the thyroid gland cannot produce thyroid hormones) ~95% of hypothyroidism is due to primary hypothyroidism SECONDARY hypothyroidism involves the pituitary-hypothalamic axis

You are seeing a 17-year old girl. All of the following would be considered risk factors for T2DM except: A) obesity B) Native American ancestry C) Family history of T1 DM D) personal history of polycystic ovary syndrome

C) family history of T1DM

When caring for a patient with DM, HTN, and persistent proteinuria, the NP should prescribe ___________. A) furosemide B) methyldopa C) fosinopril D) nifedipine

C) fosinopril

A 24-year old male walks in to the clinic c/o raccoon bite in his right thigh. On physical exam, you note a 1 cm deep wound on his right thigh that is oozing bright red blood. Your next best course of action is to A) administer high-dose parenteral penicillin B) initiate antibacterial prophylaxis with amoxicillin C) give rabies immune globlulin and rabies vaccine D) suture the wound after proper cleaning

C) give rabies immune globlulin and rabies vaccine Bats, racoons, skunks carry a significant rabies risk and should be considered rabid. Give RIG and rabies vaccine, consider tetanus

Cardiovascular effects of hyperinsulinemia include ______________. A) decreased renal sodium reabsorption B) constricted circulating volume C) greater responsiveness to angiotensin II D) diminished sympathetic activation

C) greater responsiveness to angiotensin II

Symptoms of postpartum depression include all of the following except A) overwhelming fatigue B) insomnia C) hallucinations D) severe mood swings

C) hallucinations

A risk factor for postpartum psychosis is A) history of depression B) multiple births (e.g. twins, triplets) C) history of bipolar disorder D) illegal drug use

C) history of bipolar disorder

Risk factors for postpartum depression include all of the following except A) a history of depression B) financial problems C) history of caring two or more pregnancies to term D) unplanned pregnancy

C) history of caring two or more pregnancies to term

Which of the following is the least potent topical corticosteroid? A) mometasone furoate 0.1% (Elocon ointment) B) clobetasol propionate 0.05% (Cormax) C) hydrocortisone 2.5% D) fluocinonide 0.05% (Lidex)

C) hydrocortisone 2.5 % mometasone furoate 0.1% = MIDRANGE potency clobetasol propionate 0.05% = SUPER-HIGH potency fluocinonide 0.05% (Lidex) = HIGH potency

Clinical disorders that increase the risk for osteoporosis include all of the following except A) rheumatoid arthritis B) celiac disease C) hyperlipidemia D) hyperprolactinemia

C) hyperlipidemia

In the treatment of acne vulgaris, which lesions respond best to topical antibiotic therapy? A) open comedomes B) cysts C) inflammatory lesions D) closed comedomes

C) inflammatory lesions

Metformin should be discontinued for the day of and up to 48 hours after surgery because of an increased risk of A) hypoglycemia B) hepatic impairment C) lactic acidosis D) interaction with most anesthetic agents

C) lactic acidosis

Patients with subscapular bursitis typically present with A) limited shoulder ROM B) heat over affected area C) localized tenderness under the superomedial angle of the scapula D) cervical nerve root irritation

C) localized tenderness under the superomedial angle of the scapula

The use of calcitonin to treat osteoporosis has been associated with an increased risk of A) T2DM B) rheumatoid arthritis C) malignancy D) systemic lupus erythematosus

C) malignancy Calcitonin is an antiresorptive medication that is most helpful in building vertebral bone. The FDA advises that the risks a/w calcitonin outweigh the benefits in treating osteoporosis, largely based on two studies that indicated slightly higher rates of malignancy among patients taking this agent. FDA supports the continued use of these agents as an option for patients who do not tolerate other treatments

The vitamin D needs for a 36-year old person who is taking phenytoin are best described as A) easily met by a well-balanced diet B) equivalent to what is required by other adults in this age group C) markedly increased by two- to five-fold from other adults in this age group D) reduced from baseline because of the drug's vitamin-D preserving qualities

C) markedly increased by two- to five-fold from other adults in this age group

Oral antifungal options for onychomycosis include all of the following except A) itraconazole B) fluconazole C) metronidazole D) terbinafine

C) metronidazole

The uterine fundus should be palpable at 16 weeks A) at the level of the umbilicus B) just above the level of the symphysis pubis C) midway between the symphysis pubis and umbilicus D) below the symphysis pubis

C) midway between the symphysis pubis and umbilicus The fundus should be at the level of the umbilicus at 20 weeks

In pregnant women, asymptomatic bacteruria A) is a common, benign finding B) should be treated only if bladder instrumentation or surgery is planned C) needs to be treated to avoid complicated UTI D) is a risk factor for developing hypertension

C) needs to be treated to avoid complicated UTI (acute cystitis, pyelonephritis)

As part of the evaluation of patients with OA, the NP anticipates finding A) anemia of chronic disease B) elevated CRP C) no disease-specific lab abnormalities D) elevated antinuclear antibody (ANA) titer

C) no disease-specific lab abnormalities

A 28-year old women presents to the clinic c/o cat bite on her right ankle. Her pet cat bit her when she accidentally stepped on his paw. He is 3 years old and is up-to-date on his vaccinations. She washed the wound with soap and water. Treatment of this patient's cat bite should include standard wound care with the addition of A) oral erythromycin B) topical bacitracin C) oral amoxicillin-clavulanate D) parenteral rifampin

C) oral amoxicillin-clavulanate ~80% of cat bites become infected, all wounds should be cultured and treated empirically (Only 5% of dog bites become infected, treat if severe or significant comorbidity)

Radiographic findings of OA of the knee often reveal A) microfractures B) decreased density of subchondral bone C) osteophytes D) no apparent changes to the joint structure

C) osteophytes

Pseudogout has been linked with abnormal activity of the A) liver B) kidneys C) parathyroid D) adrenal glands

C) parathyroid

The risk of infanticide is greatest in a woman with which of the following conditions A) postpartum depression B) postpartum "baby blues" C) postpartum psychosis D) There is little risk of infanticide with any of the above conditions.

C) postpartum psychosis

The meglitinide analogues (e.g. repaglinide [Prandin]) are particularly helpful adjuncts in T2DM care to minimize the risk of A) fasting hypoglycemia B) nocturnal hyperglycemia C) postprandial hyperglycemia D) postprandial hypoglycemia

C) postprandial hyperglycemia

The use of NSAIDs during pregnancy can potentially increase the risk for A) premature birth B) neural tube defects C) premature closure of the ductus arteriosis D) ventricular septal defects

C) premature closure of the ductus arteriosis

Extracorporeal shock-wave therapy can be used in the treatment of epicondylitis as a means to A) improve ROM B) build forearm strength C) promote the natural healing process D) stretch the extensor tendon

C) promote the natural healing process

In caring for a patient with OA of the knee, you advise that A) straight-leg raising should be avoided B) heat should be applied to painful joints after exercise C) quadriceps-strengthening exercises should be performed D) physical activity should be avoided

C) quadriceps-strengthening exercises should be performed

What is the most effective protection against shingles? A) having had the chickenpox as a child B) a previous episode of shingles C) receipt of the varicella-zoster immunization D) avoiding children and daycare centers

C) receipt of the varicella-zoster immunization

A patient presents with painful, blistering thermal burns on his right hand involving the thumb, index, and middle fingers. The most appropriate plan of care is to A) apply an anesthetic cream to the area and open the blisters B) apply silver sulfadiazine (Silvadene) to the area followed by a bulky dressing C) refer the patient to a burn specialist D) wrap the burn loosely with a nonadherent dressing and prescribe an analgesic

C) refer the patient to a burn specialist

Common signs of high-grade aortic stenosis in an individual during exercise include all of the following except A) dyspnea B) angina C) seizure D) syncope

C) seizure

Which of the following is the major difference between nonpurulent cellulitis and erysipelas? A) erythema B) rapid progression C) sharp and well-demarcated borders D) S. aureus

C) sharp and well-demarcated borders

A 22-year old female wants to know if she can start a walking program. She has been diagnosed with mitral valve prolapse, and her echo revealed trace mitral regurgitation. You respond that A) she should have an exercise tolerance test B) an ECG should be obtained C) she may proceed in the absence of symptoms of activity intolerance D) running should be avoided

C) she may proceed in the absence of symptoms of activity intolerance

In counseling women about SSRI use during pregnancy, the NP considers that studies reveal A) a clear teratogenic pattern has been identified for all drugs in this class B) SSRIs have a negative effect on intellectual development C) the use of paroxetine (Paxil) during pregnancy is associated with an increased risk for congenital cardiac defect D) an increased rate of seizure disorder in exposed offspring

C) the use of paroxetine (Paxil) during pregnancy is associated with an increased risk for congenital cardiac defect

Which of the following are recommended treatment options for psoriasis of the scalp? Select all that apply. A) oral antihistamines B) selenium sulfide shampoo C) topical steroids D) tar/salicylic acid shampoo

C) topical steroids D) tar/salicylic acid shampoo

ACOG and AMA recommend routing screening of all pregnant women for domestic violence at every prenatal visit. Which characteristic below is not suggestive of domestic violence? A) improbable injury B) missed prenatal visits C) unsupportive partner D) seeking prenatal care late (after 14 weeks)

C) unsupportive partner ...as opposed to an overprotective partner

Osteoporosis prevention measures include all of the following except A) calcium supplementation B) selective estrogen receptor modulator use C) vitamin B6 supplementation D) weight-bearing and muscle-strengthening exercise

C) vitamin B6 supplementation

Clinical findings of the knee in a patient with OA include all of the following except A) coarse crepitus B) joint effusion C) warm joint D) knee often locks or a pop is heard

C) warm joint

Differentiation between gout and pseudogout can involve all of the following diagnostic approaches except A) analysis of minerals in the blood B) analysis of joint fluid C) xray of the affected joint D) measuring thyroid function

C) xray of the affected joint

In caring for a patient with DM, micoralbuminuria measurement should be obtained A) annually if urine protein is present B) periodically in relation to glycemic control C) yearly D) with every diabetes-related office visit

C) yearly

Put the following in the correct order for COPD prescribing strategy: A) Long-acting anticholinergic or LABA, plus rescue med B) Inhaled corticosteroid +LABA or LA anticholinergic, plus rescue med C) Short-acting anticholinergic or SABA PRN D) Inhaled corticosteroid +LABA and/or LA anticholinergic, plus rescue med

C, A, B, D 1. Short-acting anticholinergic or SABA PRN THEN 2. Long-acting anticholinergic or LABA, plus rescue med THEN 3. Inhaled corticosteroid +LABA or LA anticholinergic, plus rescue med THEN 4. Inhaled corticosteroid +LABA and/or LA anticholinergic, plus rescue med

A pregnant mother is Rh negative. When should she receive RhoGam (anti-D immune globulin)? Select all that apply. A) At diagnosis of pregnancy B) At 12-14 weeks gestation C) At about 28 weeks gestation D) Once in each trimester E) Within 72 hours of delivery

C, E C) At about 28 weeks gestation E) Within 72 hours of delivery

How do the following conditions affect Hgb/Hct? (e.g. Increase, Decrease, Neutral) COPD: CKD: HTN: DM w/ AIC 14: Aspirin use: Testosterone use: Resident of Denver, CO: Age > 80:

COPD: Increase (tissues are chronically deprived of oxygen) CKD: Decrease HTN: Neutral DM w/ AIC 14: Decrease (poorly controlled disease, strains RBCs and causes them to die early) Aspirin use: Neutral Testosterone use: Increase Resident of Denver, CO: Increase (r/t high altitude) Age > 80: Decreased (bone marrow is not as robust...RBC production is decreased, but DO NOT assume that anemia in an older patient is due to aging!)

A 19-year old college student (otherwise healthy, nonsmoker) was diagnosed with community-acquired pneumonia by CXR a couple days ago. She has been taking amoxillin with clavulanate 875 mg BID for the past 48 hours. She returns today for a follow-up appointment. Her vitals 2 days ago: BP 120/72, HR 96, T 103F, RR 24/min, Ó2 sats 92%. Her vitals today: BP 130/80, HR 100, T 102.2, RR 24/min, Ó2 sats 94%. How would you manager her today? A) Repeat CXR, CBC, and start levofloxacin B) Start azithromycin 5-day pack C) Continue with amoxicillin-clavulanate for another 24 hours D) Stop amoxicillin-clavulanate and start doxcycline 100 mg BID x 7 days

D) Stop amoxicillin-clavulanate and start doxycycline 100 mg BID x 7 days She most likely has an atypical pathogen (mycoplasma pneumoniae or chlamydophila pneumoniae), which should be treated with a macrolide or doxycycline

A patient who takes fosinopril for HTN has been diagnosed with ACE-inhibitor cough. Which of the following statements is true? A) He could switch to lisinopril B) This cough is more likely in patients with lower airway disease C) His cough should improve over time D) The cough is related to an inability to break down bradykinin

D) The cough is related to an inability to break down bradykinin

Recommended calcium intake for a woman during pregnancy is ______________ mg of elemental calcium per day. A) 400-600 B) 600-800 C) 800-1000 D) 1000-1300

D) 1000-1300 Increased clacium intake is important for the development of bones/teeth

In a pregnant woman with asthma, in what part of her pregnancy do symptoms and bronchospasm often worsen? A) 6 to 14 weeks B) 15 to 23 weeks C) 24 to 33 weeks D) 29 to 36 weeks

D) 29 to 36 weeks Sx worsen because of esophageal irritation from GERD.

In a pregnant woman with asthma, bronchospasm symptoms are often reported to improve during ________ weeks of gestation. A) 8 to 13 weeks B) 20 to 26 weeks C) 29 to 36 weeks D) 36 to 40 weeks

D) 36 to 40 weeks Sx usually improve late in gestation when gradual fetal descent occurs

You see a 46-year old female diagnosed with vitamin D deficiency and a 25(OH)D level of 18 ng/ml. Treatment should be initiated with which of the following vitamin D regimens? A) 400 IU twice a day B) 1000 IU daily C) 10,000 IU twice a week D) 50,000 IU weekly

D) 50,000 IU weekly (for at least 8 weeks) Vitamin D3 is the preferred form of the micronutrient to treat vitamin D deficiency. A dose of 50,000 IU of vitamin D3 by mouth once per week for at least 8 weeks is advised.

Mr. Johnson is a 62-year old T2DM with an Ā1C of 7.3%. He has been started on low-dose Metformin. Over the last few weeks he has tolerated increasing doses and now takes 1000 mg twice daily. What would you expect his Ā1C to be when you recheck it in 3 months? A) 7.0% B) 6.8% C) 6.5% D) 6.0%

D) 6.0% Metformin 1000 mg BID can produce a 1-2% drop in Ā1C

Hemoglobin Ā1C best provides information on glucose control over the past A) 1-29 days B) 21-47 days C) 48-63 days D) 64-90 days

D) 64-90 days

Up to what percent of patients with medial epicondylitis recover without surgery? A) 35% B) 50% C) 75% D) 95%

D) 95%

What percentage of fetus are in vertex position by the 36th week of pregnancy? A) 30% B) 50 % C) 75% D) 95%

D) 95%

Which of the following classes of medications is commonly recommended as part of first-line therapy in a newly diagnosed T2DM? A) alpha-glucosidase inhibitor B) Meglitinide C) TZD D) Biguanide

D) Biguanide (Metformin)

The most common site for cervical radiculopathy is: A) C3 to C4 B) C4 to C5 C) C5 to C6 D) C6 to C7

D) C6 to C7

A 24-year old woman was diagnosed with iron deficiency anemia 4 weeks ago (Hgb 11.5, Hct 34.5%). Today her Hgb is 12.8 and her Hct is 38.4%. How do you proceed? A) Stop iron supplementation since she is no longer anemic B) Order a reticuloctye count C) Order a TIBC D) Consider continued iron supplementation for another 3-5 months

D) Consider continued iron supplementation for another 3-5 months (to replace iron stores)

a 58-year old female w/ hypothyroidism is taking 88 mcg of synthroid daily. She has a routine TSH with her annual labs. Today her TSH was 1.4 mU/L (normal 0.5-4.5) and T4 was 2.5 (normal 0.8-1.8). What action is appropriate? A) Increase her dose to 100 mcg daily B) Increase her dose to 112 mcg daily C) Decrease her dose D) Continue the same dose

D) Continue the same dose Her TSH is normal

Which of the following should be periodically monitoring when using a biguanide? A) CK B) ALP C) ALT D) Cr

D) Cr

Which of the following joints is most likely to be affected by OA? A) wrists B) elbows C) metacarpophalangeal (MCP) joints D) distal interphalangeal (DIP) joints

D) DIP joints

All of the following can cause an elevated maternal alpha-fetoprotein (AFP) except A) underestimated gestational age B) open neural tube defect C) meningomyelocele D) Down syndrome

D) Down syndrome Increased AFP a/w open NTDs Decreased AFP a/w Trisomy 18 and 21

Mr.Smith is a 75-year old man with an Ā1C of 8.9%. Which of the following should be the provider's first action? A) Start metformin B) Discuss exercise, weight loss C) Establish an Ā1C goal of < 7% D) Establish a target Ā1C goal

D) Establish a target Ā1C goal

An obese 55-year old woman with a history of moderate persistent asthma has a temperature of 101F, bilateral wheezes, mild shortness of breath, and purulent sputum. Her med list includes: fluticasone/salmeterol BID, albuterol PRN, amlodipine 5 mg, levothyroxine 99 mcg daily, and metformin 1000 mg BID. She is being treated for pneumonia with levofloxacin and nebulized albuterol q4-6 PRN for wheezing. She develops white plaques on the buccal mucosa, palate, and tongue (thrush). What medications are the most likely cause of this? A) Levofloxacin and metformin B) Albuterol and levofloxacin C) Metformin and fluticasone D) Fluticasone and levofloxacin

D) Fluticasone and levofloxacin

Gram-negative bacteria that commonly cause burn wound infections include all of the following except A) Pseudomonas aeruginosa B) Escherichia coli C) Klebsiella pneumoniae D) Haemophilus influenzae

D) Haemophilus influenzae

Leo is an 18-year old male who has been taking isotretinoin (Accutane) to treat his acne for the last 2 months. Which of the following is the most important question for the NP to ask at his follow-up office visit? A) Are you having any problems remembering to take your medication? B) Have you noticed any dry skin around your mouth since you started taking Accutane? C) Do you notice any improvement in your skin? D) Have you noticed any recent changes in your mood?

D) Have you noticed any recent changes in your mood?

The use of which of the following medications has the potential for causing the greatest reduction in Ā1C? A) A biguanide B) A TZD C) A sulfonylurea D) Insulin

D) Insulin

A 24-year old, otherwise healthy college student presents with c/o cough x 6 weeks. She has tried several OTC cough meds with no improvement. What is the most important information to consider when building your differential diagnoses? A) Her age B) Family hx C) Ineffectiveness of OTC cough medicines D) Length of time she has been coughing

D) Length of time she has been coughing Why? This information helps you build your ddx Acute cough < 3 weeks: bronchitis, sinusitis, PND, exacerbation of COPD/asthma, pneumonia, pulmonary embolism Chronic cough (>8 weeks) GERD and Asthma are most common causes, also consider infection (e.g. pertussis, atypical pneumonia), ACE inhibitors, chronic bronchitis, bronchiectasis, lung ca)

Mr. Johnson follows up for his BP 4 weeks after increasing his amlodipine from 5 mg to 10 mg. He is currently taking 20 mg lisinopril/25 mg HCTZ qAM and 10 mg amlodipine qAM. His average BP is 132/76. He complains of lower extremity edema and requests a change in his medications because of this. Which choice is a good next step? A) Lisinopril 20/HCTZ 25 mg qAM, change amlodipine 10 mg to qHS B) Lisinopril 20/HCTZ 25 mg BID C) Lisinopril 20/HCTZ 25 mg, decrease amlodipine to 5 mg daily, and add furosemide 10 mg daily D) Lisinopril 40 mg, HCTZ 25 mg, decrease amlodipine to 5 mg daily

D) Lisinopril 40 mg, HCTZ 25 mg, decrease amlodipine to 5 mg daily

First-line treatment options for bursitis usually include A) corticosteroid bursal injections B) heat to area C) weight-bearing exercises D) NSAIDs

D) NSAIDS

Mr. Jones is a 66-year old black man. He is a retired postal worker. PMHx includes HTN, a-fib, diabetes, HLD, gout, BPH, osteoarthritis in his knees. He quit smoking 5 years ago. His BP today is 160/90, HR 84. BMI 32. He is at increased risk for AAA. What is the preferred way to screen him for aortic aneurysm? A) Palpation of the abdomen B) Plain film of the abdomen C) Abdominal CT scan D) One-time ultrasound now What finding might you identify on physical exam?

D) One-time ultrasound now May identify a pulsatile abdominal mass on physical exam

A 45- year old patient was diagnosed with hypothyroidism 12 weeks ago. 12 weeks ago: TSH 14.3 (normal 0.4-4.8). On Levothyroxine 50 mcg 6 weeks ago: TSH 5.4. On 75 Levothyroxine 75 mcg Today: TSH 0.4 How shoulder her levothyroxine dose be managed today? A) Refer to endocrinology B) Decrease levothyroxine to 50 mcg daily C) Discontinue levothyroxine D) Resume levothyroxine 50 mcg daily and have her take 2 tabs on Saturdays/Sundays

D) Resume levothyroxine 50 mcg daily and have her take 2 tabs on Saturdays/Sundays 50 mcg daily x 7 days = 350 mcg weekly (not enough, patient was not at goal after 6 weeks) 75 mcg daily x 7 days = 525 mcg weekly (too much her TSH decreased to 0.4 after 6 weeks) 50 mcg M-F = 250 mcg 100 mcg Sat/Sun = 200 mcg 250+200 = 450 mcg weekly

A 26-year old woman has been taking an SSRI for depression during the entire course of her pregnancy. She gives birth to a full-term healthy baby girl. Five days after the delivery, she reports that the baby is irritable with protracted periods of crying. This is likely the result of A) increased intracranial pressure from en utero SSRI exposure B) colic C) impending sepsis D) SSRI withdrawal

D) SSRI withdrawal

What medication combination is considered unsafe in a patient with asthma? A) Fluticasone and albuterol B) Mometasone, formoterol, albuterol C) Budesonide and levalbuterol D) Salmeterol and levalbuterol Why?

D) Salmeterol and levalbuterol Salmeterol is a long-acting beta agonist. LABAs MUST be combined with an inhaled corticosteroid (e.g. Advair, Breo, Symbicort)

A patient with asthma symptoms multiple times throughout the day and nighttime awakenings on most nights of the week has A) Intermittent asthma B) Mild persistent asthma C) Moderate persistent asthma D) Severe persistent asthma

D) Severe persistent asthma

All of the following uropathogens are capable of reducing urinary nitrates to nitrites except A) Escherichia coli B) Proteus species C) Klebsiella pneumoniae D) Staphylococcus saprophyticus

D) Staphylococcus saprophyticus

An obese 55-year old woman with a history of moderate persistent asthma has a temperature of 101F, bilateral wheezes, mild shortness of breath, and purulent sputum. Her med list includes: fluticasone/salmeterol BID, albuterol PRN, amlodipine 5 mg, levothyroxine 99 mcg daily, and metformin 1000 mg BID. How should she be managed today? A) Treat with azithromycin daily for 5 days B) Treat with ciprofloxacin, nebulized albuterol q4-6 hours PRN for wheezing C) Treat with oral steroid and nebulized levalbuterol q8 hours PRN D) Treat with levofloxacin, nebulized albuterol q4-6 PRN for wheezing

D) Treat with levofloxacin, nebulized albuterol q4-6 PRN for wheezing She is more likely to have Strep pneumoniae and should be treated with a quinolone. Ciprofloxacin is not a respiratory quinolone

A 29-year old female reports a sudden onset of right sided facial asymmetry. She is unable to close her right eyelid tightly, frown, or smile on the affected side. Her physical exam is otherwise unremarkable. This likely represents paralysis of which cranial nerve? A) III B IV C) V D) VII

D) VII Acute facial nerve paralysis (Bell's palsy) is a possible neurologic manifestation in stage 2 Lyme disease that warrants careful examination and serologic testing for Lyme disease

In which patient is a physiologic murmur most likely? A) a 70-year old with CHF B) a 45-year old with HTN C) a 30-year old marathon runner D) a 16-year old athlete

D) a 16-year old athlete

The use of a TZD (e.g. rosiglitazone [Avandia]) is not recommended in all of the following clinical scenarios, except A) a 57-year old male taking a nitrate B) a 62-year old female with heart failure C) a 45-year old male who is on insulin D) a 35-year old patient with newly diagnosed T2DM

D) a 35-year old patient with newly diagnosed T2DM

Which of the following patients has impaired glucose tolerance? A) a 70-year old male with a fasting glucose of 109 B) an 84-year old female with a 1-hour postprandial glucose of 98 C) a 33-year old male with an Ā1C of 5.4% D) a 58-year old female with a 2-hour postprandial glucose of 152

D) a 58-year old female with a 2-hour postprandial glucose of 152

Which of the following patients is at greatest risk for developing seborrheic dermatitis? A) a 15-year old boy who lives in a rural setting? B) a 34-year old female who smokes 2 PPD of cigarettes C) a 48-year male truck driver D) a 72-year old male with Parkinson's disease

D) a 72-year old male with Parkinson's disease

You see a 28-year old Italian-American man who participates in recreational basketball and has mild hypertension. In considering treatment, you consider all of the following as viable options except A) a calcium channel blocker B) an angiotensin converting enzyme inhibitor C) an angiotensin receptor antagonist D) a beta-adrenergic antagonist

D) a beta-andrenergic antagonist

Criteria for the diagnosis of T2DM includes: A) classic symptoms, regardless of fasting plasma glucose measurement B) plasma glucose level of 126 mg/dL as a random measurement C) a 2-hour glucose measurement of 156 mg/dl after a 75-gram anhydrous glucose load D) a plasma glucose level of 126 mg/dl or greater after an 8-hour fast on more than one occasion

D) a plasma glucose level of 126 mg/dl or greater after an 8-hour fast on more than one occasion

Which of the following findings is often found in a person with stage 1 Lyme disease? A) peripheral neuropathic symptoms B) high-grade AV heart block C) Bell's palsy D) a single, painless annular lesion

D) a single, painless annular lesion (erythema migrans) Stage 1 = early, localized disease; mild flu-like sx, single annular lesion with central clearing; s/sx resolve in ~3-4 weeks

Prenatal assessment for aneuploidy should be offered to A) women > 35 years of age only B) women < 21 years of age only C) women < 21 years of age or >35 years of age only D) all women regardless of age

D) all women regardless of age

Which of the following is a diagnostic test? A) cell free fetal DNA test B) serum alpha-fetoprotein C) serum inhibin-A D) amniocentesis

D) amniocentesis

Benzodiazepine withdrawal syndrome is best characterized as A) bothersome but not life-threatening B) potentially life-threatening C) most often seen with medications with a longer half life D) associated with seizure risk

D) associated with seizure risk

Clinical manifestations of vitamin D deficiency include all of the following except A) rickets B) osteomalacia C) antigravity muscle weakness D) azotemia

D) azotemia (high levels of nitrogen-containing compounds in the blood, e.g. urea, creatine)

Pseudogout is caused by the formation of what type of crystals in joints? A) uric acid B) calcium oxalate C) struvite D) calcium pyrophosphate dihydrate

D) calcium pyrophosphate dihydrate

Regarding the current scientific evidence on the use of glucosamine and chondroitin for the management of symptomatic OA of the knee, American Academy of Orthopaedic Surgeons (AAOS) A) strongly favors their use B) provides a moderate-strength recommendation for their use C) cannot recommend for or against the use of these supplements (limited evidence) D) cannot recommend the use of these supplements

D) cannot recommend the use of these supplements

Patients with lateral epicondylitis typically present with A) an electric-like pain elicited by tapping over the median nerve B) reduced joint ROM C) pain that is worse with elbow flexion D) decreased hand grip strength

D) decreased hand grip strength

Patients with medial epicondylitis typically present with A) forearm numbness B) reduction in ROM C) pain on elbow flexion D) decreased hand grip strength

D) decreased hand grip strength

Among young adults with an implantable cardioverter defribillator (ICD), participation in sports should be A) generally cautioned against B) limited to non-contact sports C) limited to anaerobic activities D) determined on an individual basis

D) determined on an individual basis

In an older adult with T2DM and gastroparesis, the use of which of the following medications should be avoided? A) insulin glargine (Lantus) B) insulin aspart (Aspart) C) glimeperide (Amaryl) D) exenatide (Byetta)

D) exenatide (Byetta)

Acanthosis nigricans is commonly noted in all of the following areas except A) groin folds B) axilla C) nape of the neck D) face

D) face

Risk factors for lateral epicondylitis include all of the following except A) repetitive lifting B) playing tennis C) hammering D) gout

D) gout

All of the following are risks for lactic acidosis in individuals taking metformin, except A) the presence of chronic renal insufficiency B) acute dehydration C) recent use of radiographic contrast dye D) history of allergic reaction to sulfonamides

D) history of allergic reaction to sulfonamides

Women at high risk for aneuploidy include all of the following except A) maternal age of 35 years or older at delivery B) history of previous pregnancy with trisomy C) fetal ultrasound findings indicating increased risk of aneuploidy D) history of multiparity

D) history of multiparity

Which of the following statements is not true concerning the effects of exercise and insulin resistance? A) Approximately 80% of the body's insulin-mediated glucose uptake occurs in the skeletal muscle B) With regular aerobic exercise, insulin resistance is reduced by approximately 40% C) The insulin resistance-reducing effects of exercise persist for 48 hours after the activity D) hyperglycemia can occur as a result of aerobic exercise

D) hyperglycemia can occur as a result of aerobic exercise

Metformin has all of the following effects except _______________. A) improved insulin-mediated glucose uptake B) modest weight loss with initial use C) enhanced fibrinolysis D) increased LDL cholesterol production

D) increased LDL cholesterol production

The cornerstone controller therapy for moderate persistent asthma during pregnancy is the use of A) oral theophyilline B) mast cell stabilizers C) leukotriene receptor antagonists D) inhaled corticosteroids

D) inhaled corticosteroids

A mother brings her 3-year old daughter to the clinic with dry, red patches on her face around her eyes. MOP has observed her daughter constantly rubbing the area, which has caused swelling around her eyes. Physical examination is consistent with atopic dermatitis. The NP considers that this diagnosis A) requires a skin culture to confirm contibuting bacterial organisms B) should be supported by a biopsy of the affected area C) requires a peripheral blood eosinophil level D) is usually a clinical diagnosis

D) is usually a clinical diagnosis

The MOA of glucosamine and chondroitin is A) via increased production of synovial fluid B) through improved cartilage repair C) via inhibition of the inflammatory response D) largely uknown

D) largely unknown

In treating a pregnant women with acute bacterial rhinosinusitis, the NP would likely avoid prescribing A) amoxicillin B) cefuroxime C) cefpodoxime D) levofloxacin

D) levofloxacin

Which of the following is not an example of antibiotic that is used for moderate to severe pustular acne? A) doxycycline B) erythromycin C) minocycline D) levofloxacin

D) levofloxacin

Secondary causes of hyperglycemia potentially include the use of all of the following medications, except A) high dose niacin B) systemic corticosteroids C) high dose thiazide diuretics D) low dose angiotensin receptor blockers

D) low dose angiotensin receptor blockers

Initial preparticipation screening of athletes should include A) a resting 12-lead ECG B) an echocardiogram C) both a resting 12-lead ECG and an echocardiogram D) neither a resting 12-lead ECG nor echocardiogram

D) neither a resting 12-lead ECG nor echocardiogram

Among surgical and procedural interventions, American Academy of Orthopaedic Surgeons (AAOS) strongly recommends which of the following for management of symptomatic OA of the knee? A) intraarticular corticosteroid injection B) hyaluronic acid injections C) arthroscopy with lavage and/or debridement D) none of the above

D) none of the above

Tophi are best described as A) ulcerations originating on swollen joints B) swollen lymph nodes C) abscesses with one ore more openings draining pus onto the skin D) nontender, firm nodules located in soft tissue

D) nontender, firm nodules located in soft tissue

The NP prescribes a topical medication and wants it to have maximum absorption. Which of the following vehicles should the NP choose? A) gel B) lotion C) cream D) ointment

D) ointment

American Academy of Orthopaedic Surgeons (AAOS) strongly recommends all of the following therapeutic agents for the management of symptomatic OA of the knee except A) oral NSAIDs B) topical NSAIDs C) tramadol D) opioids

D) opioids

A 52-year old female walks in to the clinic c/o rat bite while trying to open a dumpster. On physical exam, you note a 1 cm deep wound that is oozing bright red blood. In addition to standard wound care, treatment of this patient should include A) rabies immune globulin B) rabies vaccine C) oral ciprofloxacin D) oral amoxicillin-clavulanate

D) oral amoxicillin-clavulanate Rabies prophylaxis is not indicated in rat bites. Tx w/ amoxicillin-clavulanate 875 mg/125 mg BID (alternative tx: doxcycline)

You are examining a 19-year old male who is diagnosed with mitral regurgitation. When he asks about participation in sports activities, you counsel him that A) participation in sports should not be affected by his condition B) he should refrain from any activities that require physical exertion C) participation will depend on the degree of atrial atrophy D) participation will depend on the degree of ventricular enlargement

D) participation will depend on the degree of ventricular enlargement

In a 22-year old female using isotretinoin (Accutane) therapy, the NP ensures follow-up to monitor for all of the following tests except A) liver enzymes B) triglyceride level C) pregnancy test D) platelet count

D) platelet count

During a preparticipation cardiovascular history, all of the following questions should be included except A) past detection of a heart murmur B) excessive, unexplained, and unexpected shortness of breath C) prior occurrence of exertional chest pain/discomfort D) prior NSAID use

D) prior NSAID use

Components of the antenatal "quad screen" include all of the following except A) serum alpha-fetoprotein (AFP) B) human chorionic gonadotropin (hCG) C) unconjugated estriol D) progesterone

D) progesterone The quad screen includes maternal serum AFP, hCG, inhibin-A, and unconjugated estriol to screen for increased risk for trisomy 21 (Down syndrome), trisomy 18 (Edwards syndrome), or open neural tube defects

Your 66-year old patient has pitting of her nails and itchy, red plaques with silvery scales on her elbows and knees. What condition does she likely have? A) actinic keratoses B) allergic contact dermatitis C) atopic dermatitis (eczema) D) psoriasis

D) psoriasis

Mr. Jones is a 66-year old black man. He is a retired postal worker. PMHx includes HTN, a-fib, diabetes, HLD, gout, BPH, osteoarthritis in his knees. He quit smoking 5 years ago. His BP today is 160/90, HR 84. BMI 32. Meds: Irbesartan 150 mg/HCTZ 12.5 mg, Tamulosin (Flomax) 0.8 mg daily, amlodipine 5 mg daily, metoprolol 50 mg daily, aspirin 81 mg daily, naproxen 500 mg PRN for knee pain, glipizide ER 5 mg (inconsistently - it sometimes makes him feel "weak") If you increase Mr. Jones' irbesartan to 300 mg daily, which lab value is the LEAST important to monitor? A) GFR B) BUN, Cr C) Potassium D) Serum glucose

D) serum glucose

Likely sequelae of intrabursal corticosteroid injuection include A) irreversible skin atrophy B) infection C) inflammatory reaction D) soreness at the site of injection

D) soreness at the site of injection

Tetracyline antibiotics (e.g. minocycline, doxycycline) should not be used in children younger than 11 years old due to the risk of ____________________. A) allergic reaction B) tendon rupture C) secondary infection D) staining of permanent teeth

D) staining of the permanent teeth

The most common locations for tophi include all of the following except A) the auricles B) the elbows C) the extensor surface of the hands D) the shoulders

D) the shoulders

Shingles most commonly involves the dermatomes of the A) legs/pubic area B) face C) upper arms/shoulders D) thorax

D) thorax

Which of the following is an oral antimicrobial option for the treatment of a community-acquired methicillin-resistant S. aureus skin infection? A) amoxicillin B) dicloxacillin C) cephalexin D) trimethoprim-sulfamethoxazole

D) trimethoprim-sulfamethoxazole

Recommended exercises for patients with OA of the hip include all of the following except A) stretching exercises of the gluteus muscles B) straight-leg raises without weights C) isometric exercises of the iliopsoas and gluteus muscles D) weight-bearing aerobic exercises

D) weight-bearing aerobic exercises

These are Mr. Jones' lipid panel results Total cholesterol 210 mg/dl LDL 130 mg/dl HDL 28 mg/dl Glucose 165 mg/dl Triglycerides 323 mg/dl What is considered first-line to treat Mr. Jones' elevated triglycerides? What medication is most important to prescribe for his abnormal lipids? A) Pravastatin B) Fenofibrate C) Fish oil D) Niacin

First-line = lifestyle modifications Medication for abnormal lipids = A) Pravastatin Mr. Jones is a diabetic, between 40-75 years of age with LDL between 70-189 mg/dl --> moderate-intensity statin for primary prevention (30-49% reduction)

Name an inhaled steroid:

Fluticasone (Flovent) Budesonide (Pulmicort) Mometasone (Asmanex)

Name a inhaled combined long-acting beta-agonist/corticosteroid

Fluticasone/salmeterol (Advair) Fluticasone/vilanterol (Breo) Budesonide/formoterol (Symbicort) Mometasone/frmoterol (Dulera)

Mr. Smith, an 80-year old smoker, has stage II COPD. Based on his medications, what is the most predictable drug-disease interaction? Losartan 50 mg, HCTZ 12.5 mg, Amlodipine 5 mg daily, Tamsulosin (Flomax) 0.8 mg daily, Atorvastatin (Lipitor) 10 mg daily, Albuterol inhaler 2 puffs PRN for SOB, tiotropium (Spiriva) once daily A) Glaucoma B) Frequent urination C) Anxiety D) Pruritis E) Hyperglycemia F) Fatigue G) Constipation

G) Constipation Amlodipine (CCB) and tiotropium (short-acting anti-cholinergic)

Which diabetes medications are associated with weight loss?

GLP-1 agonists - exenatide (Byetta, Bydureon), liraglutide (Victoza) -AND- SGLT 2 inhibitors - canagliflozin (Invokana), dapagliflozin (Farxiga)

Mr. Johnson is a 38-year old chef. He comes to the clinic for his initial visit. His vitals today are BP 150/95, P 78, RR 16, T 98.6, BMI 31. He takes Lisinopril 20mg/HCTZ 25 mg daily, Advair 250/50 BID and albuterol PRN for asthma. What is his BP goal (according to JNC 8 guidelines)? ____________ What could be done to help Mr. Johnson reach his BP goal? A) Add 5 mg amlodipine daily B) Add 50 mg losartan daily C) Add metoprolol 25 mg daily D) Continue on current medication regimen

His BP goal is < 140/90 A) Add 5 mg amlodipine daily

Mr. Jones is a 66-year old black man. He is a retired postal worker. PMHx includes HTN, a-fib, diabetes, HLD, gout, BPH, osteoarthritis in his knees. He quit smoking 5 years ago. His BP today is 160/90, HR 84. BMI 32. Meds: Irbesartan 150 mg/HCTZ 12.5 mg, Tamulosin (Flomax) 0.8 mg daily, amlodipine 5 mg daily, metoprolol 50 mg daily, aspirin 81 mg daily, naproxen 500 mg PRN for knee pain, glipizide ER 5 mg (inconsistently - it sometimes makes him feel "weak") According to JNC 8, what is his BP goal? Which of the following are possible alternatives to his current BP regimen. Select all that apply. A) Increase irbesartan to 300 mg B) Increase HCTZ to 25 mg C) Increase amlodipine to 10 mg D) Increase metoprolol to 100 mg

His BP goal is < 140/90 (all ages, diabetes present, no history of CKD) A, C, & D A) Increase irbesartan to 300 mg (ACEI for renal protection in diabetes) C) Increase amlodipine to 10 mg (black, older adults, diabetics respond well to CCBs) D) Increase metoprolol to 100 mg (NOT first choice) If you increase Mr. Jones' metoprolol to 100 mg, which of his diseases will be affected? Answer: his diabetes. Beta Blockers may mask symptoms of hypoglycemia.

What is Naegele's Rule to estimate the due date?

LMP subtract 3 months + 7 days +1 year

When the iron count is HIGH, the TIBC (total iron binding capacity) is _________

Low

The patient has the following CBC results: Is the patient anemic? RBC 4.0 HGB 11.5 HCT 35.6% MCV 90 MCH 25.7 MCHC 33.4 RDW 14.6 PLT 265 MPV 7.1

Yes

An 80-year old female c/o shortness of breath when she walks up an incline. On exam, you hear a soft murmur near the 2nd intercostal space to the right of the sternum during systole. Is this a significant murmur? Why______________ What systolic murmur is likely? A) Aortic regurgitation B) Mitral valve prolapse C) Mitral stenosis D) Aortic stenosis

Yes, this is a significant murmur because the patient experiences SOB with activity D) Aortic stenosis

What is the treatment for CAP caused by Mycoplasma pneumoniae?

Macrolide OR doxycycline

What antibiotics are avoided in CAP caused by Strep pneumo due to high rates of resistance?

Macrolides

Which of the RBC indices tells you about RBC size? What level is normal? What level indicates microcytic (small) RBCs? What level indicates macrocytic (large) RBCs?

Mean Corpuscular Volume (MCV) = Hct/RBC Normal = 80-96 Microcytic <80 Macrocytic >96

Which of the RBC indices tells you about the RBC color? What does a low level tell you about the RBC color?

Mean corpuscular hemoglobin (MCH) Low MCH = hypochromic (pale) RBC

Anemia is a reduction in one or more of what RBC measurements?

RBC count (4.2-4.9 mil/microL), hemoglobin (12-15 g/dl), or hematocrit (37-51%)

What is the treatment for CAP caused by Strep pneumo?

Respiratory quinolone (e.g. Levofloxacin, moxifloxacin, gemifloxacin) OR high-dose amoxicillin OR amoxicillin with clavulanate

Name an inhaled long-acting beta agonists (LABAs):

Salmeterol (Serevent)

According to GOLD, what is required to establish the diagnosis of COPD?

Spirometry (FEV1/FVC ratio < 70%)

What are the three most common bugs in community-acquired pneumonia?

Streptococcus pneumoniae Mycoplasma pneuomiae (atypical pathogen) Chlamydophila pneumoniae (atypical pathogen)

Which diabetes medications are associated with weight gain?

Sulfonylureas - glipizide (Glucotrol), glyburide (DiaBeta), glimeperide (Amaryl) -AND- Insulin

A patient has hypothyroidism. Her last TSH was 2.5. She takes her levothyroxine in the AM on an empty stomach. What will happen to her TSH if she swtiches to a GENERIC form of levothyroxine?

TSH could go up, or down, or stay the same...

A patient has hypothyroidism. Her last TSH was 2.5. She takes her levothyroxine in the AM on an empty stomach. What will happen to her TSH if she takes two pills instead of one?

TSH will DECREASE

A patient has hypothyroidism. Her last TSH was 2.5. She takes her levothyroxine in the AM on an empty stomach. What will happen to her TSH if she takes the levothyroxine with her morning coffee?

TSH will INCREASE

A patient has hypothyroidism. Her last TSH was 2.5. She takes her levothyroxine in the AM on an empty stomach. What will happen to her TSH if she takes the levothyroxine with an OTC PPI?

TSH will INCREASE (PPIs increase gastric acid and affect absorption)

A patient has hypothyroidism. Her last TSH was 2.5. She takes her levothyroxine in the AM on an empty stomach. What will happen to her TSH if she takes the levothyroxine after dinner?

TSH will INCREASE (due to decreased absorption of levothyroxine)

A 48-year old postmenopausal woman with T2DM diagnosed 6 weeks ago c/o persistent diarrhea with Metformin. Her Ā1C is 9.2% (goal < 7%) 1) What is the primary prescribing strategy? 2) What medication should you avoid in this patient?

1) Dual therapy (Ā1C > 9%) 2) TZDs (Actos, Avandia) increase the risk of fracture, avoid in postmenopausal women

A 77-year old female with newly diagnosed T2DM and Ā1C of 9.5%. 1) What is her Ā1C goal? 2) What occurrence should be avoided if at all possible? 3) What age-related prescribing strategy would you consider in this patient with an Ā1C of 9.5%?

1) < 8% 2) Hypoglycemia 3) Monotherapy (older adult patient)

82-year old male with T2DM is currently on max dose of Metformin. His Ā1C is 7.9%. 1) What is your plan at this visit for his Ā1C?

1) A reasonable Ā1C goal for his age is <8%. Review lifestyle modifications and decrease carb intake

A 60-year old mane with T2DM takes Metformin and Glipizide. His Ā1C is 10.2 (goal < 7%). The NP decides to start basal insulin. 1) What should be done with Metformin and glipizide? 2) How much insulin would you start this patient on?

1) Continue Metformin. STOP glipizide. 2) Start at 0.1-0.2 units/kg/day or 10 units

Match the antibiotics with the correct drug class: 1. Sulfonamide 2. Tetracycline 3. Macrolide 4. Beta-lactam A. Doxycyline B. Azithromycin C. Penicillins D.Trimethoprim-Sulfamethoxazole E. Cephalosporin F. Clarithromycin

1. Sulfonamide - D.Trimethoprim-Sulfamethoxazole (Bactrim) 2. Tetracycline - A. Doxycycline 3. Macrolide - B & F, Azithromycin and Clarithromycin 4. Beta-lactam - C & E, PCN and cephalosporins

Match the following at-risk ethnic groups to the correct genetically-based conditions. 1. Tay-Sachs disease 2. Cystic fibrosis 3. Sickle cell trait A) Northern European ancestry B) Ashkenazi Jewish ancestry C) African ancestry

1. Tay-Sachs disease = B) Ashkenazi Jewish ancestry 2. Cystic fibrosis = A) Northern European ancestry 3. Sickle cell trait = C) African ancestry

L-thyroxine (synthetic T4) is given PO daily in the morning on an empty stomach. The usual replacement dose is based on ideal body weight. How much L-thyroxine would you start a healthy, young adult?

1.6 mcg/kg/day (e.g. 120 lbs = 55 kg --> replacement 88 mcg) Levothyroxine tabs: 25 mcg, 50, 75, 88, 100, 112, 125, 137, 150, 175, 200, 300

The ADA recommends screening for the entire population over the age of _____________ every ____ years if the screening is normal.

45; 3

What is the molecular weight requirement for a drug to be unable to pass through the placental barrier? A) > 250 daltons B) > 500 daltons C) > 1000 daltons D) > 5000 daltons

> 1000 daltons

In new onset iron deficiency anemia, the RDW (red cell distribution width) is ___________.

> 15%

What are the only 2 inhaled short-acting beta agonists (SABAs):

Albuterol and levalbuterol

For a woman who was normotensive before 20 weeks gestation, an indication of preeclampsia is BP > ______. A) 130/80 mm Hg B) 140/90 mm Hg C) 150/95 mm Hg D) 160/100 mm Hg

B) 140/90

How many grams of a topical cream or ointment are needed for a single application to the hands? A) 1 B) 2 C) 3 D) 4

B) 2

In teaching a patient with T2DM about using rapid-acting insulin to help with the managemen of postprandial hyperglycemia, the NP advises the patient that the usual dose is ____ unit(s) per 15 grams of carbohydrates. A) 1 B) 2 C) 3 D) 4

B) 2

Approximately ___ % of all clinically recognized pregnancies end in spontaneous abortion A) 10 B) 20 C) 30 D) 40

B) 20%

Which precursor of vitamin D is the form that is commonly measured in lab tests to determine vitamin D status? A) vitamin D2 B) 25-hydroxyvitamin D C) vitamin D3 D) 1, 25-dihydroxyvitamin D

B) 25-hydroxyvitamin D 25(OH)D - considered the most accurate functional indicator of vitamin D stores, the results of this test are minimally influenced by recent dietary intake or recent sun exposure

If prescribing fluconazole, the NP should consider that the drug is a cytochrome P-450 A) 3À4 inhibitor B) 2CP inhibitor C) 2D6 inducer D) 1Á2 inducer

B) 2CP inhibitor (Itraconazole is a CYP-450 3À4 inhibitor; Terbinafine has significantly less drug interaction potential)

Recommended length of antimicrobial therapy for a pregnant woman with asymptomatic bacteruria is A) 1 to 3 days B) 3 to 7 days C) 7 to 10 days D) 2 weeks

B) 3 to 7 days

Generally, testing for T2DM in asymptomatic, undiagnosed individuals over the age of 45 years, should be done every ______________. A) year B) 3 years C) 5 years D) 10 years

B) 3 years

How many grams of a topical cream or ointment are needed for a single application to the entire body? A) 10 to 30 B) 30 to 60 C) 60 to 90 D) 90 to 120

B) 30 to 60

For a woman with a healthy pre-pregnancy BMI, daily caloric requirements during pregnancy are typical baseline caloric needs plus _____ kcal. A) 100 B) 300 C) 600 D) 1000

B) 300

During pregnancy, the most intense organogenesis occurs how many days after the LMP? A) 12-30 days B) 31-81 days C) 92-120 days D) 121-150 days

B) 31 - 81 days Before day 31 post-LMP, the pregnancy exists as a group of poorly differentiated cells with no discrete organ systems. After day 81, organs are formed but still growing/developing.

In the first few weeks of a viable intrauterine pregnancy, serum hCG levels usually double every ___ hours until approximately 10,000-20,000 mIU/mL A) 24 B) 48 C) 72 D) 96

B) 48 hours

Approximately what percentage of patients with xray findings of OA of the knee will report having symptoms? A) 25% B) 50% C) 70% D) 90%

B) 50%

Consideration should be given to setting an Ā1C goal in a 22-year old male with an 8-year hx of T1DM who does not have comorbid conditions at equal to or less than ___________. A) 5.5% B) 6% C) 6.5% D) 7%

B) 6%

The daily amount of vitamin D3 recommended for pregnant/lactating women is A) 400 IU B) 600 IU C) 1000 IU D) 1200 IU

B) 600 IU

You examine a patient with red, tender thermal burn that blanches easily and involves the entire surface of the anterior left leg. The estimated involved body surface area (BSA) is approximately _________________. A) 5% B) 9% D) 13% C) 18%

B) 9%

What is the molecular weight requirement for a drug to easily pass through the placental barrier? A) < 250 daltons B) < 500 daltons C) < 1000 daltons D) < 5000 daltons

B) < 500 daltons

What antibiotics are avoided in CAP caused by atypical pathogens?

Beta-lactams (ineffective)

Name 3 causes of anemia. Which is the most common reason in the US?

Blood loss (most common), sick bone marrow, increased RBC destruction 1) Blood loss - melena, hematemesis, trauma 2) Bone marrow does not make enough RBCs -not enough iron, folate, B12 -bone marrow disorders (e.g. aplastic anemia)-bone marrow suppression (e.g. chemo) -low levels of erythropoietin (e.g. chronic renal failure) -anemia of inflammation (e.g. malignancy, anemia of chronic disease) 3) Increased RBC destruction -inherited disorders (e.g. thalassemia, sickle cell anemia) -malaria -hemolytic anemia (e.g. G6PD deficiency)

Stage 3 Lyme disease, characterized by joint pain and neuropsychiatric symptoms, typically occurs how long after the initial infection? A) 1 month B) 4 months C) 1 year D) 5 years

C) 1 year

What is the daily calcium recommendation for women over the age of 50? A) 800 mg B) 1000 mg C) 1200 mg D) 1500 mg

C) 1200 mg + 800 to 1000 IU of Vitamin D daily for all adults over the age of 50

The "quad screen" should be obtained at about _____ weeks of pregnancy. A) 6 to 10 B) 11 to 15 C) 16 to 20 D) 21 to 25

C) 16 to 20

Edwards syndrome is the clinical manifestation of trisomy ____. A) 13 B) 15 C) 18 D) 21

C) 18 Quad Screen: Decreased AFP, decreased hCG

Osteoporosis is defined as having a bone density more than _____ standard deviation(s) below the average bone mass for women < 35 years of age. A) 1 B) 1.5 C) 2.5 D) 4

C) 2.5 Dual-energy xray absoptiometry (DEXA) is the preferred screening test for osteoporosis

A patient has a fundal height measurement of 24 cm. She is probably about A) 14-18 weeks gestational age B) 18-22 weeks gestational age C) 22-26 weeks gestational age D) In her 3rd trimester

C) 22-26 weeks gestational age

What is the recommended weight gain during pregnancy for a woman with a healthy pre-pregnancy BMI? A) 15-20 lbs B) 20-30 lbs C) 25-35 lbs D) 35-45 lbs

C) 25-35 lbs

How many grams of a topical cream or ointment are needed for a single application to an arm? A) 1 B) 2 C) 3 D) 4

C) 3

A child must consume ____ oz. of fortified milk each day to receive the recommended 400 IU daily of vitamin D. A) 8 B) 16 C) 32 D) 48

C) 32 oz

There is good evidence in support of oral steroids for COPD exacerbations to shorten recovery time and improve lung function. What is the correct recommended dose? A) Medrol dose-pack B) 10-day course of Prednisone 20 mg, followed by a taper C) 5-day course of Prednisone 40 mg

C) 5-day course of Prednisone 40 mg Chronic use should be avoided - associated with an unfavorable risk-to-benefit ratio

In a healthy person, what percentage of the body's total daily physiological insulin secretion is released basally? A) 10% B) 25% C) 50% D) 75%

C) 50%

For a woman with a healthy pre-pregnancy BMI, daily caloric requirements during lactation are typical baseline caloric needs plus _____ kcal. A) 200 B) 300 C) 500 D) 1000

C) 500

When is a patient with an ectopic pregnancy most likely to present for evaluation? A) 2-4 weeks after conception B) 4-6 weeks after conception C) 6-8 weeks after LMP D) 8 or more weeks after LMP

C) 6-8 weeks after LMP

Approximately ___% of spontaneous abortions are associated with chromosomal defects. A) 20 B) 40 C) 60 D) 80

C) 60%

For adults under the age of 70 years old, what is the recommended daily intake of vitamin D? A) 200 IU B) 400 IU C) 600 IU D) 1000 IU

C) 600 IU

The duration of antimicrobial therapy for the treatment of symptomatic UTI in a pregnant woman is A) 3 days B) 5 days C) 7 days D) 10 days

C) 7 days Treatment options for asymptomatic bacteruria and symptomatic UTI include beta lactams (e.g. amoxicillin, cephalexin, cefpodoxime, cefixime, and amoxicillin/clavulanate) and nitrofurantoin *Nitrofurantoin should be avoided after the 36th week of gestation because of potential risk for hemolysis if the fetus is G6PD deficient

The recommended Ā1C goal in a 79-year old female with a 20-year hx of T2DM who has difficulty ambulating, uses a walker, and has a history of heart failure with a cardiac ejection fraction of 35% should be equal to or less than __________________. A) 7% B) 7.5% C) 8% D) 8.5%

C) 8%

Which of the following should be periodically monitoring when using a thiazolidinedione? A) CK B) ALP C) ALT D) Cr

C) ALT

Mr. Jones (a 66-year old black man. He is a retired postal worker. PMHx includes HTN, a-fib, diabetes, HLD, gout, BPH, osteoarthritis in his knees) has had 2 episodes of heart failure in the past 12 months, one that required in-patient hospitalization. He is managed by a cardiologist who instructed him to take furosemide 40 mg daily for 3 days when he began to experience an "overnight weight increase and shortness of breath." He has done this with minimal improvement in shortness of breath. Cardiology can't see him until tomorrow morning, but they told him to see his PCP or go to the ER if his symptoms worsen. He does not want to go to the ER. He presents to the clinic today c/o feeling tired, mildly short of breath, with a cough, and knee pain. His vitals today: BP 125/80, P 104, RR 22, T 98.2 Meds: Irbesartan 300 mg daily, HCTZ 25 mg daily, Furosemide 40 mg daily PRN 2-lb weight gain in 2 days, metoprolol 100 mg daily, pravastatin 40 mg daily, glipizide 5 mg/500mg metformin daily, flomax daily, aspirin, naproxen What diagnostic test would be most useful in determining the cause of his heart failure? A) Blood glucose B) UA C) EKG D) chest xray What specific abnormality would you be looking for?

C) EKG A-fib Early signs of HF = S3 and/or S4, resting tachycardia, tachypnea, crackles Peripheral edema is a later sign

Five years or more after T2DM diagnosis, which of the following medications is less likely to be effective in controlling plasma glucose? A) Metformin B) Pioglitazone C) Glipizide D) Insulin

C) Glipizide

Type I hypersensitivity reactions, such as atopic dermatitis, involve the action of which antibodies binding to receptor sites on mast cells? A) IgG B) IgM C) IgE D) IgA

C) IgE

The most common sites for lumbar disk herniation are: A) L1 to L2 and L2 to L3 B) L2 to L3 and L4 to L5 C) L4 to L5 and L5 to S1 D) L5 to S1 and S1 to S2

C) L4 to L5 and L5 to S1

Loss of posterior tibial reflex often indicates a lesion at A) L3 B) L4 C) L5 D) S1

C) L5

With the straight-leg test, the NP is evaluating tension on which of the following nerve roots? A) L1 and L2 B) L3 and L4 C) L5 and S1 D) S2 and S3

C) L5 and S1

Loss of Achilles tendon reflex most likely indicates a lesion at: A) L1 to L2 B) L3 to L4 C) L5 to S1 D) S2 to S3

C) L5 to S1

A patient's hemoglobin is 10.2 and hematocrit is 30.6%. Which findings are consistent with an iron deficiency anemia? A) MCV 76, MCH 28 B) MCV 84, MCH 26 C) MCV 75, MCH 25 D) MCV 120, MCH 30

C) MCV 75 (low), MCH 25 (low) Microcytic, hypochromic anemia

According to the CDC, what drug class is considered first-line treatment for pertussis? A) Sulfonamide B) Tetracycline C) Macrolide D) Beta-lactam

C) Macrolide antiobitic (e.g. Azithromycin, clarithromycin Sulfonamides are second-line

A 30-year old male has persistent asthma. What daily medication regimen would be appropriate? A) Albuterol B) Low-dose fluticasone, albuterol C) Medium-dose fluticasone D) Budseonide, salmeterol, albuterol

C) Medium-dose fluticasone Albuterol alone is used for intermittent asthma Albuterol should not be used daily (if the patient is using their rescue inhaler more than twice a week --> call the PCM!)

A nurse practitioner auscultates a murmur at the apex of the heart. What systolic murmur is likely? A) Mitral stenosis B) Aortic stenosis C) Mitral regurgitation D) Aortic regurgitation

C) Mitral regurgitation

A patient with asthma symptoms daily with occasional nighttime awakenings has A) Intermittent asthma B) Mild persistent asthma C) Moderate persistent asthma D) Severe persistent asthma

C) Moderate persistent asthma

Mr. Jones, a 45-year old drug and alcohol counselor, smokes 1 PPD and c/o cough, low-grade fever, and night sweats for the last week. His CXR show bilateral hilar nodes. What should you do next? A) Refer to Pulmonology B) Order a chest CT with contrast C) Order a TB skin test D) Repeat the CXR in 2 weeks

C) Order a TB skin test

A patient who complains of fatigue has a TSH of 13.4 (normal 0.4-4.8 mU/ml). What should be done next? A) Order a TSH next week B) Order a thyroid panel C) Repeat the TSH and add a free T4 D) Prescribe levothyroxine

C) Repeat the TSH and add a free T4

Which medication combination presents the lowest risk for hypoglycemia to a patient? A) Metformin + NPH insulin B) Metformin + sulfonylurea C) TZD + DPP-4 D) SGLT2 + basal insulin

C) TZD + DPP-4

Among individuals < 35 years of age, the most common cause of sudden cardiac death is A) non-ST segment MI B) ST-segment MI C) a congenital cardiac malformation D) aortic stenosis

C) a congenital cardiac malformation

The mechanism of action of metformin (Glucophage) is as A) an insulin-production enhancer B) a product virtually identical in action to sulfonylureas C) a drug that increases insulin action in the peripheral tissues and reduces hepatic glucose production D) a facilitator of renal glucose excretion

C) a drug that increases insulin action in the peripheral tissues and reduces hepatic glucose production

You examine a 24-year old pregnant woman with a history of mild intermittent asthma at 24 weeks gestation for an acute asthma flare. Her medication regimen should be adjusted to include A) titration to a therapeutic theophylline level B) addition of timed salmeterol (Serevent) use C) a short course of oral prednisone D) use of montelukast (Singulair) on a regular basis

C) a short course of oral prednisone

The American Academy of Orthopaedic Surgeons (AAOS) favors all of the following in the management of symptomatic OA of the knee except A) low-impact aerobic exercise B) weight loss for those with a BMI of 25 or greater C) acupuncture D) strengthening exercises

C) acupuncture

Treatment of pseudogout can include all of the following except A) NSAIDs B) colchicine C) allopurinol D) oral corticosteroids

C) allopurinol

All of the following medications are recommended for the treatment of concomitant HTN in patients with T2DM, except A) beta blockers B) calcium channel blockers C) alpha adrenergic receptor blockers D) angiotensin receptor blockers

C) alpha adrenergic receptor blockers

The mechanism of action of pioglitazone is as: A) an insulin-production enhancer B) a reducer of pancreatic glucose output C) an insulin sensititizer D) a facilitator of renal glucose excretion

C) an insulin sensititizer pioglitazone (Actos) is a thiazolidinedione

What are the side effects associated with anticholinergic medications?

Cognitive impairment, confusion, hallucinations, dry mouth, blurry vision, urinary retention, constipation, tachycardia, acute angle glaucoma "Can't see, can't pee, can't spit, can't shit."

What class of diabetes medications is considered weight neutral?

DPP-4 inhibitors "gliptins" sitagliptin (Januvia), linagrliptin (Tradjenta)

Mrs. Smith is 85 years old and lives alone. She drives herself and participates in chair aerobics for 30 minutes, 3 days a week and does supervised resistance training twice a week. Her BP today is 160/70. BMI 23. PMHx includes extensive CAD with 2 stents placed 5 years ago, HTN, osteopenia, hyperlipidemia, GERD, hypothyroidism, and osteoarthritis. Meds: Pravastatin, Lisinopril 10 mg/HCTZ 12.5 mg daily, Aspirin 81 mg, Omeprazole 20 mg, Levothyroxine 25 mcg daily, Naproxen PRN for OA. Her BP record 2 weeks ago, 170/80, 12 days ago 175/78, 10 days ago 168/94, 1 week ago 170/85, 3 days ago 174/88, yesterday 172/80. Her labs reveal evidence of renal insufficiency ( Cr 1.6, eGFR 45 ml/min) According to JNC 8 guidelines, what is her BP goal? How could her HTN be managed today? A) Add amlodipine 2.5 mg daily B) Increase lisinopril/HCTZ to 20/12.5 mg daily C) Add losartan 25 mg daily D) Increase HCTZ to 25 mg

Her BP goal is < 150/<90 A) Add amlodipine 2.5 mg daily Mrs. Smith has isolated systolic HTN = significant predictor of cardiovascular events in older adults

When the iron count is LOW, the TIBC (total iron binding capacity) is _________

High

Name a short-acting inhaled anticholinergic:

Ipratropium (Atrovent)

Name a inhaled combined short-acting anticholinergic/short-acting beta agonist:

Ipratropium/albuterol (Combivent)

In ____________ iron deficiency anemia, the RDW is < 15%.

Longstanding, chronic (all RBCs are small)

Iron deficiency aemia is a _______cytic, ______chromic anemia. What is the most common cause?

Microcytic, hypochromic Blood loss is the most common cause (Patients are usually asymptomatic until H/H < 10/30)

A 67-year old male complains of pain in his left lower leg when he walks. He gets complete relief when he stops walking and rests. What 2 findings are not likely? A) Weakened left pedal pulse B) Left lower extremity edema C) Presence of CAD D) Absence of hair growth on lower legs E) History of smoking F) History of diabetes G) History of hypothyroidism

Not likely findings in peripheral artery disease B) Left lower extremity edema = more likely in patients with venous insufficiency G) History of hypothyroidism What are the two biggest risk factors for PAD? Smoking and diabetes

Name a long-acting inhaled anticholinergic:

Tiotropium (Spiriva)


संबंधित स्टडी सेट्स

module 6 review test- STATISTICS

View Set

Econ 102 Sample Test Chapter 3 Spring 2020

View Set

Explicit, Implicit, Imply, Infer, Inferences

View Set

Practice Test: Module 07 Network Architecture

View Set

Ex3 Peds LWW Toddler Flash Cards FA20

View Set

Pharmacognosy and Plant chemistry (VIOLET)

View Set